63
Midyear Exam KSMC (2014) 100 QS (senior) Choose the best answer: 1- A 16-year-old boy suffers a mid-gut volvulus, taken to surgery, small bowel resection done, about 46 cm left from ileum. Which of the following statements is false concerning his nutritional requirements and management? A. The patient may tolerate some form of enteral nutrition. B. A nutritional regimen consisting of supplemental glutamine, growth hormone, and a modified high carbohydrate, low fat diet may be beneficial in this patient. C. A nutritional regimen consisting of supplemental glutamine, growth hormone, and a modified high carbohydrate, high fat diet may be beneficial in this patient. D. The suitable regimen described may decrease the cost of care. Answer: C In selected patients, however, with residual small intestine (at least 18 inches), post- resectional hyperplasia may develop with time such that they can tolerate enteral feeds. Recent studies have demonstrated the requirement for TPN could be decreased or even eliminated in patients with short-gut syndrome by providing a nutritional regimen consisting of supplemental glutamine, growth hormone, and a modified high carbohydrate, low fat diet. There was a marked improvement in absorption of nutrients with this combination of therapy and a decrease in stool output. In addition, TPN requirements were reduced by 50% as were costs associated with the care of these individuals. Discontinuation of the growth hormone did not increase TPN needs in patients once they had undergone successful gut rehabilitation. 2- A 17-year-old patient involved in RTA is paralyzed with multiple peripheral extremity injuries. Nutritional support is instituted with a transnasal feeding catheter. Which of the following statements is false concerning the patient’s management? A. Feeding into the stomach results in stimulation of the biliary/pancreatic axis, which is probably trophic for small bowel. B. Gastric secretions will dilute the feedings increasing the risk of diarrhea. C. The major risk in this patient is tracheobronchial aspiration. D. Placement of the feeding catheter through the pylorus into the first portion of the duodenum reduces the risk of regurgitation and aspiration. Answer: B The use of transnasal feeding catheters for intragastric feeding or for duodenal intubation are popular adjuncts for providing nutritional support by the enteral route. The stomach is easily accessed by passage of a soft flexible feeding tube. Intragastric feeding provides several advantages for the patient. The stomach has the capacity and reservoir for bolus feedings. Feeding into the stomach results in

Q and a Exam for Seniors in general surgery

Embed Size (px)

DESCRIPTION

Q and a Exam for Seniors in general surgery

Citation preview

Page 1: Q and a Exam for Seniors in general surgery

Midyear Exam KSMC (2014) 100 QS (senior) Choose the best answer: 1- A 16-year-old boy suffers a mid-gut volvulus, taken to surgery, small bowel resection done, about 46 cm left from ileum. Which of the following statements is false concerning his nutritional requirements and management?

A. The patient may tolerate some form of enteral nutrition. B. A nutritional regimen consisting of supplemental glutamine, growth

hormone, and a modified high carbohydrate, low fat diet may be beneficial in this patient.

C. A nutritional regimen consisting of supplemental glutamine, growth hormone, and a modified high carbohydrate, high fat diet may be beneficial in this patient.

D. The suitable regimen described may decrease the cost of care.

Answer: C In selected patients, however, with residual small intestine (at least 18 inches), post-resectional hyperplasia may develop with time such that they can tolerate enteral feeds. Recent studies have demonstrated the requirement for TPN could be decreased or even eliminated in patients with short-gut syndrome by providing a nutritional regimen consisting of supplemental glutamine, growth hormone, and a modified high carbohydrate, low fat diet. There was a marked improvement in absorption of nutrients with this combination of therapy and a decrease in stool output. In addition, TPN requirements were reduced by 50% as were costs associated with the care of these individuals. Discontinuation of the growth hormone did not increase TPN needs in patients once they had undergone successful gut rehabilitation.

2- A 17-year-old patient involved in RTA is paralyzed with multiple peripheral

extremity injuries. Nutritional support is instituted with a transnasal feeding

catheter. Which of the following statements is false concerning the patient’s

management?

A. Feeding into the stomach results in stimulation of the biliary/pancreatic axis, which is probably trophic for small bowel.

B. Gastric secretions will dilute the feedings increasing the risk of diarrhea. C. The major risk in this patient is tracheobronchial aspiration. D. Placement of the feeding catheter through the pylorus into the first portion

of the duodenum reduces the risk of regurgitation and aspiration.

Answer: B

The use of transnasal feeding catheters for intragastric feeding or for duodenal

intubation are popular adjuncts for providing nutritional support by the enteral

route. The stomach is easily accessed by passage of a soft flexible feeding tube.

Intragastric feeding provides several advantages for the patient. The stomach has

the capacity and reservoir for bolus feedings. Feeding into the stomach results in

Page 2: Q and a Exam for Seniors in general surgery

stimulation of the biliary/pancreatic axis which is probably trophic for the small

bowel. Gastric secretions will have a dilutional effect on the osmolarity of the

feedings, reducing the risk of diarrhea. The major risk of intragastric feeding is the

regurgitation of gastric contents resulting in aspiration into the tracheobronchial

tree. This risk is highest in patients who have an altered sensorium or who are

paralyzed. The placement of the feeding tube through the pylorus into the fourth

portion of the duodenum reduces the risk of regurgitation and aspiration of feeding

formulas.

3-Products of platelet degranulation include all except:

A. Tumor necrosis factor. B. Alpha granules degranulation. C. Transforming growth factor b. D. Platelet-derived growth factor. E. Dense granules degranulation.

Answer: A

The initial response to injury and disruption of a blood vessel is bleeding. The

hemostatic response to this is clot formation to stop hemorrhage. Platelet plug

formation initiates the hemostatic process along with clotting factors activated by

collagen and the basement membrane proteins exposed by the injury. Platelets then

degranulate, releasing the contents of their alpha granules and dense granules,

most notably platelet derived growth factor and transforming growth factor b. These

substances initiate chemotaxis and proliferation of inflammatory cells, beginning

the inflammatory response that will ultimately heal the wound. Tumor necrosis

factor and interleukin-1 also stimulate fibroblast proliferation, however are

produced by macrophages.

4- Which of the following surgical techniques lead to improved wound healing?

A. Traumatic handling of tissue , and meticulous hemostasis. B. Approximation of underlying fatty tissue to obliterate dead space. C. Protecting the wound from water for at least one day. D. Using povidone\iodine components for dressing.

Answer: C

There are numerous practical implications for the care of wounds and surgical

incisions. Meticulous hemostasis reduces the inflammation of phagocytosis

necessary to clear the wound of blood. Atraumatic handling of tissue decreases the

load of necrotic or nonviable cells at the wound margin. Deep sutures are best

placed only into collagen laden structures that will hold tension, i.e., fascia and

dermis. These tissues have a tensile strength to hold sutures under tension. Fat

does not contain collagen and will not hold tension. Therefore, fatty tissue should

not be sutured as a separate layer. Given that epithelialization of an incision is

Page 3: Q and a Exam for Seniors in general surgery

normally complete within 24–48 hours, there is no reason to protect the incision

from water beyond this time. Allowing the patient to wash or shower one or two

days after surgery actually serves useful purpose in debriding the wound.

5- A 40 year old male patient brought to ER with multiple injuries post RTA, which

of the following statements about diagnostic peritoneal lavage (DPL) is true?

A. DPL is the diagnostic procedure of choice for gunshot wounds to the

abdomen with no obvious intra-abdominal injuries.

B. The average reported incidence of false-positive DPL in patients with

significant pelvic fractures is 10% to 15%.

C. Accuracy rates for DPL have generally been reported as high as 97%.

D. DPL has been entirely replaced by computed tomography as the diagnostic

procedure of choice following blunt abdominal trauma.

Answer: C

DPL remains the most sensitive and specific indicator of intra-abdominal injury in

the trauma patient. The accuracy rates for DPL in several large collective series

reveal an overall sensitivity of 95%, specificity of 98% to 99%, and overall accuracy

of 97%. As result, DPL remains the mainstay for diagnosis of intraperitoneal injury

in the trauma patient; however, not every trauma patient requires DPL. In the awake,

alert, and responsive patient with isolated abdominal injuries, the physical

examination and history are very helpful in predicting the presence of significant

injury. In the patient with lower torso (nipples to pubis) or back or flank gunshot

wounds, the incidence of intra-abdominal injury is so high that exploratory

laparotomy without further diagnostic modalities is generally advocated. In

addition, DPL is generally inaccurate in the diagnosis of retroperitoneal injuries

(duodenum, renal, pancreas), and significant retroperitoneal hemorrhage in

association with pelvic fractures produces a false-positive DPL rate of up to 30%.

Computed tomography (CT) scans have proved extremely valuable in these

situations. General recommendations for the use of abdominal CT scans in trauma

victims include patients who are hemodynamically stable (normal) with (1)

equivocal abdominal examination, (2) closed head injury, (3) spinal cord injury, (4)

hematuria, and (5) pelvic fractures with significant bleeding. These five indications

are appropriate if the patient is truly hemodynamically stable and the time required

to perform CT does not delay any surgical procedures.

6- An untreated or an unrecognized compartment syndrome produces nerve and

muscle damage and prevents good functional recovery despite the patency of

vascular repair. Which of the following factors is false about suggesting the need

for a fasciotomy?

A. A period of 6 hours or more between injury and restoration of perfusion. B. Combined arterial and venous injuries. C. Postoperative signs of loss of palpated pulse. D. Elevated compartment pressures. E. Preoperative calf swelling.

Page 4: Q and a Exam for Seniors in general surgery

Answer: C

Factors that suggest the need for fasciotomy are as follows:

1. Prolonged period (6 hours or more) between injury and restoration of

perfusion

2. Associated crush injury

3. Preoperative calf swelling

4. Combined arterial and venous injuries

5. Extensive venous ligation

6. Postoperative signs or disproportionate muscle pain, pain on passive

stretch, or tender and firm muscles

7. Elevated compartment pressures

7- Which of the following statements regarding unusual hernias is incorrect?

A. An obturator hernia may produce nerve compression diagnosed by a

positive Howship-Romberg sign.

B. Grynfeltt's hernia appears through the superior lumbar triangle, whereas

Petit's hernia occurs through the inferior lumbar triangle.

C. Sciatic hernias usually present with a painful groin mass below the inguinal

ligament.

D. Littre's hernia is defined by a Meckel's diverticulum presenting as the sole

component of the hernia sac.

E. Richter's hernia involves the antimesenteric surface of the intestine within

the hernia sac and may present with partial intestinal obstruction.

Answer: C

Sciatic hernias usually present with intestinal obstruction or a mass in the gluteal

or infragluteal region.

8- A 72-year-old man had multiple injuries and an altered sensorium after a high-speed motor vehicle collision. He was intubated for his decreased mental status. During intubation, a large amount of gastric contents was noted in the posterior pharynx and he aspirates. Which of the following is the appropriate initial treatment?

A. Bronchoscopy for aspiration of particulate matter. B. Steroids. C. Prophylactic antibiotics. D. Inhaled nitric oxide. E. High positive end-expiratory pressure.

The answer is A Bronchoscopy is indicated for aspiration pneumonitis if there is particulate matter in the tracheobronchial tree. Prophylactic antibiotics are not indicated after gross aspiration, which results in a chemical pneumonitis. Antibiotics are indicated only if pneumonia develops. Steroids and inhaled nitric oxide are not indicated for

Page 5: Q and a Exam for Seniors in general surgery

aspiration pneumonitis. High positive end-expiratory pressure is not required in patients unless respiratory failure develops.

9- A 63-year-old man with multiple rib fractures and a pulmonary contusion requires prolonged intubation. He is unable to be weaned from the ventilator and is on a volume control mode. He has a tracheostomy and a percutaneous gastrostomy in place through which he is being fed.The RQ is 1. Based on this information, which of the following is the next step in his management?

A. Decrease the inspired concentration of oxygen. B. Decrease the rate on the ventilator. C. Increase the rate on the ventilator. D. Decrease the carbohydrates in his enteral feeds. E. Increase the total number of calories in his enteral feeds.

The answer is D. The respiratory quotient (RQ) is the ratio of the rate of carbon dioxide production over the rate of oxygen uptake. An RQ of 0.75 to 0.85 is ideal. The RQ is dependent on the composition of nutritional support. An RQ of 0.7 indicates a diet comprised largely of lipids. An RQ of 1 or greater indicates a diet comprising primarily carbohydrates, which can impair weaning from the ventilator. Decreasing the carbohydrates in the enteral feeds would facilitate the patient’s ventilator wean. Changing ventilator settings by decreasing the inspired concentration of oxygen or the rate may be indicated as part of the ventilator wean, but these will not affect the RQ.

10- Following surgery a patient develops oliguria. You believe the oliguria is due to hypovolemia, but you seek corroborative data before increasing intravenous fluids. Which of the following values supports the diagnosis of hypovolemia?

A. Urine sodium of 28 mEq/L. B. Urine chloride of 15 mEq/L. C. Fractional excretion of sodium more than 1. D. Urine/serum creatinine ratio of 20. E. Urine osmolality above 500 mOsm/kg.

answer is E. A fractional excretion of sodium (FENa) is calculated as (urine sodium × serum creatinine) ÷ (serum sodium × urinary creatinine) × 100. A FENa less than 1% supports a prerenal etiology for the patient’s oliguria. When oliguria occurs postoperatively, it is important to differentiate between low output caused by the physiologic response to intravascular hypovolemia and that caused by acute tubular necrosis. A FENa of less than 1% in an oliguric setting indicates aggressive sodium reclamation in the tubules. Values above this suggest a tubular injury such that Na cannot be appropriately reclaimed. In the setting of postoperative hypovolemia, all findings would reflect the kidney’s efforts to retain volume: the urine sodium would be below 20 mEq/L,

Page 6: Q and a Exam for Seniors in general surgery

the urine chloride would not be helpful except in the metabolically alkalotic patient, the urine osmolality would be over 500 mOsm/kg, the urine/serum creatinine ratio would be above 20, and the blood urea nitrogen (BUN)/creatinine ratio would be above 20.

11- A patient with a nonobstructing carcinoma of the sigmoid colon is being prepared for elective resection. Which of the following reduces the risk of postoperative infectious complications?

A. A single prophylactic preoperative enteral dose of antibiotic effective against aerobes and anaerobes.

B. Postoperative administration for 48 hours of parenteral antibiotics effective against aerobes and anaerobes.

C. Postoperative administration of parenteral antibiotics effective against aerobes and anaerobes until the patient’s intravenous lines and all other drains are removed

D. Redosing of antibiotics in the operating room if the case lasts for more than 4 hours.

The answer is D. The appropriate dosing and timing of antibiotic prophylaxis to prevent surgical site infectionsin an elective procedure is a single dose, no greater than 1 hour prior to the incision. Additionally, most textbooks recommend use of an oral, nonabsorbable antibiotic regimen effective against aerobes and anaerobes in combination with a mechanical bowel preparation before elective colon resections. There is no evidence to support the continuation of antibiotics for more than 24 hours after an elective operation has been completed, and this practice should be avoided to prevent increasing microbial drug resistance. For complex, prolonged procedures, redosing of antibiotics may be appropriate during the procedure based on the drug’s half-life. Broad-spectrum antibiotic coverage, including against anaerobic organisms, is required only in cases where such flora are anticipated, such as during colon resections. ------------------------------------------------------------------------------------------------------------ 12- A 60-kg, 53-year-old man with no significant medical problems undergoes lysis of adhesions for a small-bowel obstruction. Postoperatively, he has high nasogastric output and low urine output. What is the most appropriate management of his fluids?

A. Infusion of D5 0.45% normal saline at 100 mL/h B. Infusion of D5 0.9% normal saline at 100 mL/h C. Infusion of D5 lactated Ringer at 100 mL/h D. Replacement of nasogastric tube losses with lactated Ringer in addition to

maintenance fluids. E. Replacement of nasogastric tube losses with 0.45% normal saline with 20

mEq/L of potassium chloride in addition to maintenance fluids. The answer is D. This postoperative patient requires replacement of nasogastric tube losses in

Page 7: Q and a Exam for Seniors in general surgery

addition to maintenance fluids. In an otherwise healthy individual, maintenance fluids are calculated based on body weight as 4 mL/kg/h for the first 10 kg, 2 mL/kg/h for the second 10 kg, and 1 mL/kg/h for every additional kg body weight. A 60-kg man requires 100 mL of fluid per hour or 2400 mL of fluid per day. Five percent dextrose in 0.45% normal saline with or without potassium chloride (depending on renal function) would be an appropriate maintenance fluid. Both lactated Ringer and normal saline, which are isotonic, can be used to replace gastrointestinal losses.

13- A 70-kg woman is to undergo nail removal from her right ring finger in the ambulatory surgery clinic. Which of the following is the most appropriate option for local anesthesia?

A. Digital block with 1% lidocaine without epinephrine up to 4.5 mg/kg. B. Digital block with 1% lidocaine with epinephrine up to 4.5 mg/kg. C. Digital block with 1% lidocaine with epinephrine up to 7 mg/kg. D. Local injection around the nail bed with 1% lidocaine with epinephrine up

to 4.5 mg/kg E. Local injection around the nail bed with 1% lidocaine with epinephrine up

to 7 mg/kg/Ml The answer is A. The maximal safe total dose of lidocaine administered to a 70-kg person is 4.5 mg/kg, or approximately 30 to 35 mL of a 1% solution. The addition of epinephrine to lidocaine, procaine, or bupivacaine not only doubles the duration of infiltration anesthesia, but increases the maximal safe total dose by one-third (eg, 7 mg/kg for lidocaine with epinephrine) by decreasing the rate of absorption of drug into the bloodstream. However, epinephrine-containing solutions should not be injected into tissues supplied by end arteries (eg, fingers, toes, ears, nose, penis). Hypersensitivity to local anesthetics is uncommon and occurs most prominently with anesthetics of the ester type (procaine, tetracaine). While small nerve fibers seem to be most susceptible to the action of local anesthetics, these agents act on any part of the nervous system and on every type of nerve fiber. CNS toxicity usually appears as stimulation followed by depression, probably because of an early selective depression of inhibitory neurons; with a massive overdose, all neurons may be depressed simultaneously.

14- Axillary lymph node dissection is routinely used for all of the following

conditions except:

A. 2-cm. pure comedo-type intraductal carcinoma.

B. 1-cm. infiltrating lobular carcinoma.

C. 8-mm. infiltrating ductal carcinoma.

D. A pure medullary cancer in the upper inner quadrant.

Answer is A

Intraductal carcinoma is carcinoma in situ and does not metastasize to regional or

distant sites. Lymph node dissection is not routinely required for a pure in situ

Page 8: Q and a Exam for Seniors in general surgery

cancer of the breast. In contrast, all of the other cancers listed above (infiltrating

lobular, infiltrating ductal, and medullary carcinoma) are invasive malignancies

that are capable of nodal and distant metastasis. Lymph node dissection is

commonly recommended for these invasive malignancies. Intraductal lesions that

have grown larger than 5 cm. are more apt to have become focally invasive. Since

this invasive component might be missed histologically, many surgeons advocate

selective use of axillary node dissection for large intraductal lesions, particularly

high-grade tumors such as the comedo variant. However, a purely intraductal 2-

cm. cancer would most likely be treated without performing node dissection.

15- Which of the following statements is correct with regard to gastric bypass for

obesity?

A. Rapid weight loss following successful gastric bypass for obesity is associated with an increased risk of developing cholelithiasis

B. Marginal ulcer develops in 25% of gastric bypass patients C. Vitamin B12 deficiency is a potential complication of gastric bypass due to

gastric mucosal atrophy D. Anastamotic leak after gastric bypass is often heralded by bradycardia

Answer: A

The most serious complication after gastric bypass for obesity is anastamotic

dehiscence. Leak is presumed to occur secondary to gastric necrosis due to

ischemia from staple line application or short gastric vessel ligation. Affected

patients may have little pain, with tachycardia, tachypnea and fever as the only

manifestations. Physical examination in morbidly obese patients with peritonitis is

unreliable. Marginal ulcers occur in only 10% or less of gastric bypass patients and

respond to H2 receptor antagonists. Rarely, polyneuropathy has been noted after

gastric bypass, usually in association with intractable vomiting and protein calorie

malnutrition. Vitamin B12 deficiency has been noted after gastric bypass due to

decreased acid digestion of vitamin B12 in food; monthly B12 supplementation

should be routine. Cholelithiasis occurs in about one third of morbidly obese

patients and gallstone formation is accelerated in the early postoperative period by

the effects of rapid weight loss.

16- Failure to perform radiation after wide excision of an invasive cancer risks

which of the following outcomes?

A. Recurrence of cancer in the ipsilateral breast.

B. Shorter survival time.

C. Regional nodal recurrence.

D. Greater chance of breast cancer mortality.

Answer is A

Retrospective reviews and prospective surgical trials agree that omission of

breast radiation after wide excision leads to a higher rate of ipsilateral breast

recurrence. However, survival and the risk of distant disease are not altered in

Page 9: Q and a Exam for Seniors in general surgery

patients treated by excision alone, within the follow-up time of the studies and

given their inherent power to detect differences in outcome. Regional node

metastasis is not affected by the choice of mastectomy versus wide excision and

radiation.

17- Which of the following statements IS FALSE concerning mammography?

A. Up to 50% of cancers detected mammographically are not palpable.

B. One third of palpable breast cancers are not detected by mammography

C. The sensitivity of mammography increases with age.

D. The American Cancer Society currently recommends routine screening

mammography beginning at age 40.

Answer is B

Although mammography has been available for years, it did not become widely

used until the findings of the Health Insurance Plan of New York and the Breast

Cancer Detection Demonstration project studies of screening mammography were

disseminated. These and other investigators demonstrated that 10%–50% of

cancers detected mammographically are not palpable. Conversely, palpation

recognizes 10%–20% of tumors not detectable mammographically. The incidence

of breast cancer begins to rise sharply at age 40, and the sensitivity of

mammograms increases with age as the dense parenchymal tissue of young

women is progressively replaced by fatty tissue. Routine screening

mammography has been shown to decrease breast cancer-related mortality in

asymptomatic women over the age of 50. Controversy exists concerning the role

of screening in younger woman. However, currently the American Cancer Society

recommends that mammographic screening begin at age 40. Although sensitive,

mammography is not specific. Only about 25% of nonpalpable lesions detected

mammographically are found to be malignant at biopsy. A spiculated density with

ill-defined margins on mammogram is almost certainly malignant. Most

commonly, features are seen that are suggestive but not diagnostic of cancer.

These include clustered microcalcifications, asymmetric density, ductal

asymmetry, and distortion of normal breast architecture and/or skin or nipple

distortion.

18- A 35-year-old woman, who is currently breast-feeding her firstborn child,

develops an erythematous and inflamed fluctuant area on breast examination.

Which of the following statement is false concerning her diagnosis and

management?

A. The most common organism which would expect to be cultured is

Staphylococcus aureus.

B. Open surgical drainage is likely indicated.

C. Breast-feeding absolutely should be discontinued.

D. If the inflammatory process does not completely respond, a biopsy may

be indicated.

Page 10: Q and a Exam for Seniors in general surgery

Answer is C

Infection complicates breast-feeding in fewer than 1:100 women, but these

lactational infections still account for 80% of all breast infections. Presumably,

gaining access via the skin of the irritated nipple of the nursing woman,

Staphylococcus aureus is by far the most common pathogen in this setting.

Many breast infections begin as cellulitis, without abscess formation. When an

actual abscess is suspected, percutaneous aspiration can establish the

diagnosis and allow for bacterial culture and sensitivity testing. Open surgical

drainage is the most prudent and effective treatment. Although women may

choose to cease breast feeding, there is no absolute indication for this. When

mastitis or breast infection is suspected clinically, the possibility of an

inflammatory carcinoma must also be entertained. Any inflammatory process

that does not respond completely and promptly to antibiotics or drainage should

be subjected to biopsy to rule out cancer.

19- A familial form of medullary thyroid carcinoma (MTC) should be suspected

whenever:

A. The tumor is unilateral.

B. The tumor is bilateral.

C. Pathologic examination of the resected thyroid gland reveals the absence

of C-cell hyperplasia in areas of the gland adjacent to foci of MTC.

D. All of the above.

Answer is B.

Sporadic MTC is unilateral in at least 80% of cases. However, in patients with

MTC occurring as a component of the multiple endocrine neoplasia (MEN) type 2A

or type 2B syndromes, the tumor is virtually always multifocal and bilateral.

Typically, in this setting the MTC appears as multiple whitish-tan tumor nodules in

the middle and upper thirds of each thyroid lobe.

A diffuse premalignant proliferation of the C cells of the thyroid is thought to

precede the development of MTC in patients with familial MTC. This proliferation,

known as C-cell hyperplasia (CCH), consists of parafollicular clusters of increased

numbers of C cells. The finding of CCH in areas of the thyroid adjacent to gross

foci of MTC is strong evidence for familial MTC.

20- Which of the following are true concerning islet cell neoplasms of the

pancreas in patients with MEN type 1?

A. Islet cell neoplasms in patients with MEN 1 are characteristically solitary.

B. The most common islet cell neoplasm in patients with MEN 1 is insulinoma.

C. Most of Islet cell neoplasms in patients with MEN 1 are malignant.

D. Islet cell neoplasms of the pancreas occur in 30% to 80% of patients with

MEN 1.

E. All of the above.

Page 11: Q and a Exam for Seniors in general surgery

Answer is D

The pathologic change in the pancreas of patients with MEN 1 is typically

multicentric. Diffuse hyperplasia of islet cells and microadenoma formation are

often identified in areas of the gland distant from grossly evident tumor. Tumors

are commonly multifocal. Islet cell neoplasms of the pancreas occur in 30% to

80% of patients with MEN 1. The most common islet cell neoplasm in these

patients is gastrinoma. Gastrinomas associated with MEN 1 probably account for

20% to 50% of all cases of the Zollinger-Ellison syndrome. The second most

common islet cell tumor is insulinoma. Other pancreatic islet cell neoplasms,

such as glucagonoma, somatostatinoma, or vasoactive intestinal polypeptide

neoplasm (VIPoma), are rarely associated with MEN 1. Approximately 10% of

insulinomas and approximately 15% or more of gastrinomas in patients with MEN

1 are malignant.

21- Hypoparathyroidism: all true except:

A. Is most commonly encountered as a postviral syndrome.

B. Can be associated with marked hypocalcemia after parathyroidectomy in

patients with bone disease.

C. Can cause anxiety, depression, or confusion.

D. Can cause physical signs such as Chvostek's and Trousseau's signs.

E. Is treatable acutely with intravenous calcium salts and chronically with oral

calcium and vitamin D.

Answer is A

Hypoparathyroidism is most commonly encountered after thyroid surgery. It can

be temporarily severe after parathyroidectomy in patients with bone disease

(“postoperative bone hunger”). The signs and symptoms can include anxiety,

depression, confusion, Chvostek's sign, and Trousseau's sign, as well as

circumoral or extremity tingling, tetany with carpopedal spasms, or seizures.

Treatment is as noted.

22- Addisonian crisis, or acute adrenocortical insufficiency: one true

A. Occurs only in patients with known adrenal insufficiency or in those

receiving long-term supraphysiologic doses of exogenous steroids.

B. Can mimic an acute abdomen with fever, nausea and vomiting, abdominal

pain, and hypotension.

C. May cause electrolyte abnormalities, including hypernatremia,

hypokalemia, hypoglycemia, and hypercalcemia, as well as eosinophilia on

peripheral blood smear.

D. Should be diagnosed with the rapid ACTH stimulation test before steroid

replacement is instituted.

Answer is B

Addisonian crisis, or acute adrenal insufficiency, may be seen following even mild

illness in patients with suppression of the hypothalamic-pituitary-adrenal axis.

Page 12: Q and a Exam for Seniors in general surgery

This suppression can be produced by as little as 1 week of supraphysiologic

stress dose steroids in the year before the stressful event. Addisonian crisis is a

medical emergency that requires prompt treatment based on clinical suspicion.

Clinical findings include fever, nausea and vomiting, abdominal pain, and

hypotension. Laboratory analysis may reveal electrolyte abnormalities, including

hyponatremia, hyperkalemia, hypoglycemia, and hypercalcemia, as well as

eosinophilia on peripheral blood smear. The rapid ACTH test is diagnostic, but it

should not delay treatment with intravenous fluid resuscitation, glucose

replacement, and high-dose dexamethasone. Dexamethasone, not

hydrocortisone, should be given initially, since it does not interfere with

subsequent determination of plasma cortisol. Stress dose steroids are inadequate

once adrenal crisis has occurred, and exogenous mineralocorticoids are given

when the patient resumes oral intake.

23- Preparation for surgical removal of a pheochromocytoma includes all except:

A. Beta-adrenergic blockade followed by alpha-adrenergic blockade.

B. Hydration.

C. Alpha-adrenergic blockade, with or without beta-adrenergic blockade.

D. D-. Planning removal through an anterior, posterior, or laparoscopic

approach based upon tumor localization with CT, magnetic resonance

imaging (MRI), and/or 131I-MIBG.

Answer is A

Principles of preoperative management for pheochromocytoma include

preoperative alpha-adrenergic blockade using phenoxybenzamine or

phentolamine. Beta-adrenergic blockade with propranolol is then used selectively

in patients who develop tachycardia, have a history of cardiac arrhythmia, or have

primarily epinephrine-secreting tumors. Beta-adrenergic blockade should be

undertaken only after successful alpha blockade is established. Patients with

pheochromocytoma frequently exhibit intravascular volume depletion, and careful

hydration is mandatory. Central venous pressure monitoring alone is helpful to

guide hydration; more intensive monitoring with a Swan-Ganz catheter is

indicated for patients with pre-existing heart disease. Formerly, the anterior

approach was preferred for adrenalectomy as it facilitated complete abdominal

exploration and search for extra-adrenal pheochromocytoma. Accurate

preoperative localization with CT, MRI, and 131I-MIBG has allowed selective use

of the posterior, or even the laparoscopic, approach for adrenalectomy.

24- A patient requires both cardiac and renal transplantation. Preparation for the procedures has begun. How do cardiac allografts differ from renal allografts?

A. Cardiac allografts are matched by HLA tissue typing and renal allografts are not.

B. Cardiac allografts can tolerate a longer period of cold ischemia than renal allografts.

C. One-year graft survival for cardiac allografts is substantially lower than that for renal allografts.

D. Cardiac allografts are matched only by size and ABO blood type.

Page 13: Q and a Exam for Seniors in general surgery

E. Cyclosporine is a critical component of the immunosuppressive regimen for cardiac allografts but not renal allografts.

The answer is d. Cardiac allograft has become an accepted treatment for end-stage heart disease. One-year cardiac allograft survival exceeds 80%, which is comparable to renal allograft survival. Cardiac allografts have a cold ischemia preservation time of 4 to 5 hours, and therefore tissue typing is not practical. Cardiac donors are matched to recipients only by size and ABO blood type.

25- A 48-year-old woman maintained on Warfarin for a history of cardiac valvular

replacement and a history of recent upper respiratory infection presents with

severe abdominal pain exacerbated by movement. Her physical examination shows

tenderness in the right paramedian area with voluntary guarding but no peritoneal

signs. The following statements true concerning the diagnosis and management of

this patient.

A. Urgent laparotomy should be performed because of concern for bowel

ischemia.

B. The correct diagnosis could likely be made by selective mesenteric CT

angiogram.

C. Anticoagulation should be started.

D. If untreated, hemodynamic instability is uncommon.

Answer: D

Rectus sheath hematoma results from arterial or venous bleeding into the rectus

sheath, most commonly from arterial bleeding. Rectus sheath hematomas

predominate in women by a ratio of about 3:1. The mean age of incidence is in the

late fifth decade. Although spontaneous formation of a rectus hematoma is rare, it

can occur with vasculitis, arterial venous malformations, a severe coagulopathy, or

with the administration of anticoagulants. The usual cause is trauma. Events as

trivial as sneezing, coughing, or twisting to the side have initiated a rectus

hematoma. Abdominal pain is almost always described at presentation. Pain is

often described as severe and usually is exacerbated by movements that require

muscular contraction of the abdominal wall. On examination, there is tenderness

over the rectus sheath, voluntary guarding, and often a diffuse mass sensation in

the area of tenderness. Contraction of the rectus muscle exacerbates the pain and

tenderness. Peritoneal signs are absent. Ecchymosis may occur but usually

appears several days after the onset of pain. In cases where the hematoma dissects

or originates inferiorly and expands into the prevessicle and preperitoneal space,

the hematocrit may fall significantly; however, hemodynamic instability is distinctly

unusual. When the intraabdominal source of pain is unknown, ultrasound and

particularly computed tomography can delineate the hematoma and localize it to

the abdominal wall in almost all cases.

Treatment must take into consideration the cause, if known, and whether the

hematoma is stable or progressive. Coagulopathy should be corrected when

Page 14: Q and a Exam for Seniors in general surgery

possible. For patients in whom the hematoma is stable, pain medication and

avoidance of muscular stress on the abdominal wall are sufficient. For patients with

progressive hematoma, the treatment of choice is evacuation of the hematoma from

within the rectus sheath and hemostasis, sometimes requiring ligation of the

epigastric vessels above and below the hematoma.

26- 30 years old man presented to ER with generalized abdominal pain, febrile and

tachycardia, she is a known case of CRF. The following statement about peritonitis

are all true except:

A. Peritonitis is defined as inflammation of the peritoneum.

B. Most surgical peritonitis is secondary to bacterial contamination.

C. Primary peritonitis has no documented source of contamination and is more

common in adults than in children and in men than in women.

D. Tuberculous peritonitis can present with or without ascites.

Answer: C

Peritonitis is inflammation of the peritoneum and can be septic or aseptic, bacterial

or viral, primary or secondary, acute or chronic. Most surgical peritonitis is

secondary to bacterial contamination from the gastrointestinal tract. Primary

peritonitis refers to inflammation of the peritoneal cavity without a documented

source of contamination. It is more common in children than in adults and in women

than in men. The female predominance is felt to be explained by entry of organism

into the peritoneal cavity through the fallopian tubes. The clinical manifestations of

tuberculous peritonitis are of two types. The moist form consists of fever, ascites,

abdominal pain, and weakness. The dry form presents in a similar manner but

without ascites.

27- True statements regarding appendiceal neoplasms include which of the

following?

A. All carcinoid tumors of the appendix less than 1 cm are adequately treated

by simple appendectomy.

B. Appendiceal carcinoma is associated with secondary tumors of the GI tract

in up to 60% of patients

C. Survival following right colectomy for a Dukes’ stage C appendiceal

carcinoma is similar to same staged colon cancer at 5 years.

D. Mucinous cystadenocarcinoma of the appendix is adequately treated by

simple appendectomy.

E. Up to 50% of patients with appendiceal carcinoma have metastatic disease,

with the liver as the most common site of spread

Answer: C

Carcinoids represent two-thirds of all appendiceal neoplasms. Nearly half of all GI

carcinoids arise in the appendix at a mean age of 41 years. Two-thirds of the time

the carcinoid is only incidentally detected, only 0.5% have evidence of distant

metastatic spread at resection. In one experience, carcinoids between 1.5 and 2.0

cm have had minimal metastatic potential and those smaller than 1.5 cm never

Page 15: Q and a Exam for Seniors in general surgery

metastasized. In the 1% that are larger than 2 cm however, metastases are frequent

and 80% recur even after resection at this size.

Adenocarcinoma of the appendix is exceedingly rare. These tumors occur in elderly

patients at the base of the appendix. Appendicitis often follows and the diagnosis

is not made preoperatively and is rarely considered during surgery since the

appearance of the tumor may mimic perforated appendicitis. Up to half the patients

have metastatic disease at diagnosis and the peritoneum is the most common site

of spread. Survival is proportional to tumor stage. Dukes’ Stage A disease may be

treated simply with appendectomy if all disease can be removed with reasonable

margins. Dukes’ B and C lesions require formal right hemicolectomy for disease

control. Survival is, stage for stage, similar to colon cancer after 5 years.

Appendiceal adenocarcinomas also appear to have an association with secondary

tumors, often of the GI tract, in up to 35% of patients.

Patients with mucinous cystadenocarcinoma of the appendix typically are

symptomatic, and wide resection of the primary disease, together with debulking of

peritoneal implants, is indicated. Indolent progression of metastases commonly

results in prolonged survival rates (50% at 5 years) during which patients may

require repeated laparatomies for complications of the disease.

28- Which of the following statements about achalasia is correct?

A. In most cases in North America the cause is a parasitic infestation by

Trypanosoma cruzi.

B. Chest pain and regurgitation are the usual symptoms.

C. Distal-third esophageal adenocarcinomas may occur in as many as 20% of

patients within 10 years of diagnosis.

D. Manometry demonstrates failure of LES contractions on swallowing and

absent or weak simultaneous contractions in the esophageal body after

swallowing.

E. Endoscopic botulinum toxin injection of the LES, pneumatic dilatation, and

esophagomyotomy provide highly effective palliative therapy for achalasia.

Answer: E

While in South America achalasia is the result of Chagas' disease caused by

parasitic infestation by the leishmanial forms of T. cruzi, in Europe and North

America the cause of achalasia is unknown. The common presenting symptoms

of achalasia are dysphagia, regurgitation, and weight loss. Chest pain is an

infrequent symptom in achalasia and is more characteristic of esophageal spasm.

Achalasia is a premalignant esophageal lesion: the retention esophagitis leads to

metaplasia and squamous cell carcinoma, which occurs after 15 to 25 years in the

middle third of the thoracic esophagus in 10% of patients. The classic manometric

findings of achalasia are failure of relaxation of the LES on swallowing and absent

or weak simultaneous contractions in the esophageal body after swallowing.

Achalasia is currently incurable, and, though the recently described endoscopic

botulinum toxin injection of the lower esophageal sphincter, pneumatic dilatation,

Page 16: Q and a Exam for Seniors in general surgery

and esophagomyotomy effectively relieve dysphagia in the majority of patients, all

of these treatments are strictly palliative. The motility disturbance persists

throughout life.

29- 40 years old male patient presented with history of early satiety and on off

vomiting, investigations showed gastric mass going with gastric lymphoma, all of

the following regarding your patient is correct except:

A. Stage I gastric lymphomas cannot be cured completely with surgical

therapy alone.

B. Extensive gastric lymphomas that initially treated with radiation and/or

chemotherapy occasionally perforate during treatment and require

secondary resection.

C. Patients explored with a presumptive diagnosis of gastric lymphoma

should undergo an attempt at curative resection when this is safe and

feasible.

D. Appropriate staging for primary gastric lymphoma includes bone marrow

biopsy.

Answer: A

Operation alone is adequate treatment for very early-stage lymphoma, although

chemotherapy is commonly added. For more advanced disease, particularly stages

III and IV, preoperative radiation chemotherapy is often indicated, even though

some of these patients suffer perforation during therapy and require emergent

resection. Patients who undergo exploration for gastric mass without a

preoperative diagnosis can safely be resected with potential for cure even if the

diagnosis includes gastric lymphoma.

30- Type I gastric ulcers are located in the gastric body, usually along the lesser

curvature. Which of the following statements correctly characterize type I gastric

ulcers?

A. Normal to high acid secretion.

B. Associated duodenal ulceration.

C. Blood group A.

D. Associated hypergastrinemia frequent.

Answer: C

Gastric ulcers are divided into categories based on their location and gastric acid

secretory status. A type I gastric ulcer is an ulcer in the body of the stomach, usually

along the lesser curvature, associated with large volumes of secretion with a low to

normal acid output. Type I ulcers are not associated with duodenal, pyloric, or

prepyloric mucosal abnormalities. There is a slight predominance of patients with

blood group A in this type of gastric ulcer.

Type II gastric ulcer is located in the body of the stomach in combination with a

duodenal ulcer. These patients are usually acid hypersecretors. About 23% to 25%

of gastric ulcers are type II. A type III gastric ulcer is characterized as a prepyloric

Page 17: Q and a Exam for Seniors in general surgery

ulcer and accounts for about 23% of lesions. Patients with this lesion are typically

acid hypersecretors.

Type IV gastric ulcer occur high on the lesser curvature near the gastroesophageal

junction. In the United States, the incidence of type IV gastric ulcer is less than 10%.

31- 70 years old male patient presented with picture of perforated visicus,

intraoperative you find perforated gastric ulcer and the size was 5 cm, Which of the

following statements is correct regarding giant gastric ulcers ?

A. Giant gastric ulcers occur in 30–40% of cases along the greater curvature.

B. The lesser curvature is the most common site, with only 3% to 4% occurring

along the greater curvature.

C. The risk of malignancy increases with giant ulcer defined as ulcer more than

5 cm.

D. The treatment of choice for giant gastric ulcer is subtotal gastrectomy to

include the ulcer.

E. Giant gastric ulcer is a complication of intraarterial hepatic chemotherapy.

Answer: B

A giant gastric ulcer is defined as an ulcer whose diameter is 3 cm or greater. The

lesser curvature is the most common site, with only 3% to 4% occurring along the

greater curvature. Gastric ulcers often penetrate into contiguous structures such

as spleen, pancreas, liver, and transverse colon, and can be falsely diagnosed as a

nonresectable malignancy, despite normal biopsy results. Most reports cite an

incidence of malignancy ranging from 6% to 30%, increasing with the size of the

ulcer. Because of the high likelihood of complications from giant gastric ulcer, early

operation is the treatment of choice. The operation of choice is resection, including

the ulcer. If the ulcer has penetrated adjacent structures and cannot be dissected

free, the stomach wall can be incised circumferentially, leaving the ulcer intact and

behind, and the gastric resection completed.

32- A 45-year-old male presents with symptoms of epigastric pain, worsened with

ingestion of food. Physical examination is normal. Upper abdominal

ultrasonography is unremarkable. Contrast radiography reveals a 2 cm ulcer in the

gastric fundus along the lesser curvature. Therapy with omeprazole 20 mg per day

is begun but symptoms persist 3 weeks later. Appropriate management includes

which of the following?

A. Increase in omeprazole dose to 40 mg per day

B. Addition of sucralfate 1 gm every 8 hours

C. Addition of cimetidine 200 mg b.i.d.

D. Esophagogastroduodenoscopy with biopsy of ulceration

Answer: D

About 5% of ulcers appearing radiographically benign are malignant} Gastroscopy

is the most reliable method of distinguishing benign and malignant gastric ulcer,

with an accuracy of more than 97% if multiple biopsies and brushings for cytology

Page 18: Q and a Exam for Seniors in general surgery

are performed. Clinical features prompting early endoscopic evaluation include

major weight loss, symptoms of gastric outlet obstruction, a palpable abdominal

mass, and stool hemoccult positivity or blood loss anemia. Endoscopic features

that suggest malignancy include an exophytic mass, abnormal or disrupted

mucosal folds, necrotic ulcer crater, bleeding from the edge of the ulcer crater, a

stepwise depression of the ulcer edge, heaped-up margins, or small extensions of

the ulcer that blur a portion of the ulcer wall. If initial biopsies do not demonstrate

malignant cells but the endoscopic appearance strongly suggests that underlying

the ulcer is a carcinoma, repeat endoscopy with deeper biopsies should be

undertaken.

33- A 55-year-old male is evaluated because of symptoms of epigastric pain and

anorexia. Physical examination is normal except for guaiac positivity of stool.

Upper endoscopic examination reveals a 1.5 cm ulcer along the lesser curvature of

the stomach proximal to the incisura angularis. Optimal management consists of

medical treatment and which of the following:

A. Endoscopic biopsy from the ulcer rim and the ulcer base.

B. Endoscopic biopsy of the ulcer rim.

C. Endoscopic cautery of the ulcer base.

D. Endoscopic biopsy of the ulcer base.

E. Endoscopic biopsy from the ulcer rim and cautery of the ulcer base.

Answer: B

The symptoms produced by gastric cancer and benign gastric ulcer are non-

specific and often similar. Pain is present in 70% of patients with gastric cancer and

is usually constant, non-radiating, and not improved by food ingestion. Physical

examination is usually normal in patients with early gastric cancer. Guaiac

positivity is noted in one third.

Fiberoptic endoscopy is the definitive diagnostic method. Although the endoscopic

appearance of gastric ulcers may suggest benign or malignant origins, definite

distinction can only be made by gastric biopsy. Accurate diagnosis of gastric

cancer can be made in 95% of cases if multiple biopsies are obtained from the ulcer

rim. Biopsy of the ulcer base will more frequently reveal necrotic material.

34- Which of the following statements is FALSE concerning the pathophysiology of

frostbite?

A. Frostbite injury may have two components: initial freeze injury and a

reperfusion injury that follows during rewarming

B. The formation of extracellular ice crystals in the tissue begins to occur at -

2°C

C. The release of oxygen free radicals and arachidonic acid metabolites

aggravates vasoconstriction and platelet and leukocyte aggregation

D. Experimental evidence suggests that a substantial component of severe

cold injury may be mediated initially due to platelet aggregation

Answer: D

Page 19: Q and a Exam for Seniors in general surgery

Recent evidence suggests that frostbite injury may have two components: the initial

freeze injury, and a reperfusion injury that occurs during rewarming. The initial

response to tissue cooling is vasoconstriction and arterio-venous shunting,

intermittently relieved by vasodilatation. With prolonged exposure, this response

fails, and the temperature of the freezing tissues will approximate ambient

temperature until -2°C. At this point, extracellular ice crystals form, and as these

crystals enlarge, the osmotic pressure of the interstitium increases resulting in

movement of intracellular water into the interstitium. Cells begin to shrink and

become hyperosmolar, disrupting cellular enzyme function.

During rewarming, red cell, platelet and leukocyte aggregation is known to occur

and results in patchy thrombosis of the microcirculation. These accumulated blood

elements are thought to release, among other products, the toxic oxygen-free

radicals and the arachidonic acid metabolites which further aggravate

vasoconstriction and platelet and leukocyte aggregation. Recent experimental

evidence suggests that a substantial component of severe cold injury may be

neutrophil-mediated in that a monoclonal antibody to neutrophil-endothelial and

neutrophil-neutrophil adherence can markedly ameliorate the pathology of severe

injury.

35- During the evolution of the understanding of hematologic diseases, the

indications for splenectomy have changed. The most common indications for

splenectomy are, in descending order of frequency:

A. Traumatic injury, immune thrombocytopenia, hypersplenism.

B. Immune thrombocytopenic purpura, traumatic injury, hypersplenism.

C. Hypersplenism, traumatic injury, immune thrombocytopenia.

D. Immune thrombocytopenia, hypersplenism, traumatic injury.

E. None of the above.

Answer: A

Over the years, the indications for splenectomy have varied as better understanding

and altered therapy for immune system diseases have occurred. Whereas in the

past splenectomy was the available therapy for hypersplenism, better

understanding of the diseases causing hypersplenism (which is a syndrome) has

allowed other therapies to emerge. Splenectomy, therefore, is less often required

for the treatment of hypersplenism associated with some types of hematologic

malignancies. Traumatic injury remains the most common indication for

splenectomy, although this situation may change as conservative management of

splenic injuries continues to increase. Moreover, immune thrombocytopenia is

increasing in frequency and is definitively cured by splenectomy.

36- Thrombotic thrombocytopenic purpura (TTP) is a syndrome characterized by all

of the following except:

A. Afebrile thrombocytopenia and microangiopathic hemolytic anemia.

B. Deposition of platelet microthrombi.

C. Fluctuating neurologic abnormalities.

D. Renal failure with survival rate less than 10%.

Page 20: Q and a Exam for Seniors in general surgery

E. Called Moschcowitz's syndrome and the best treatment is plasmapheresis.

Answer: A

This disease, sometimes called Moschcowitz's syndrome, is characterized by

thrombocytopenia, microangiopathy, chemolytic anemia, fluctuating neurologic

abnormalities, progressive renal failure, and fever. Platelet deposits, with hyaline

material composed of aggregated platelets and fibrin, occur. The cause is unknown,

and the prognosis is very poor: survival is less than 10%. A combined approach

using antiplatelet drugs and corticosteroids can be effective and sometimes has

improved results if done with splenectomy.

37- You are consulted regarding a 50-year old male with Laennec’s cirrhosis, portal

hypertension and hypersplenism. He has no history of gastrointestinal bleeding.

You would recommend which of the following?

A. Splenectomy.

B. Prophylactic sclerotherapy for esophageal varices.

C. Portosystemic shunt.

D. Observation.

Answer: D

Patients with hypersplenism from portal hypertension usually present with

thrombocytopenia as the primary manifestation. The role of splenectomy in such

patients is controversial. Although splenectomy may transiently increase the

number of platelets in the peripheral circulation, it generally does not improve

survival. At the time of surgery, these patients have considerable risk for massive

bleeding and subsequent complications related to hepatic dysfunction. These latter

complications include ascites, coagulopathy, and wound complications. The

severity of the underlying liver disease is the determining factor for these patients.

Some believe that splenectomy is contraindicated in patients with portal

hypertension. Indeed, the perioperative risks are considerably greater than normal.

This should be considered as a last option, and even then with the realization that

the operative and preoperative morbidity and mortality are high.

In the absence of gastrointestinal bleeding, this particular patient has no indication

for either sclerotherapy or portosystemic shunting.

38- Lateral aberrant thyroid is (A) abnormally descended thyroid tissue in the lateral neck (B) associated with thyroglossal duct cyst (C) cervical lymph node metastases from papillary cancer (D) cervical lymph node enlargement as a result of Hashimoto’s disease (E) asymmetric goiter protruding predominantly to one side of the neck Page

Answer is C Lateral aberrant thyroid is cervical lymph node metastases from a papillary carcinoma of the thyroid. Papillary carcinoma most commonly presents as a

Page 21: Q and a Exam for Seniors in general surgery

thyroid nodule. Some lesions may be subclinical. Despite their small size, these lesions may demonstrate metastases. Up to 80% of papillary cancers will be associated with lymph node metastases; however, most are microscopic and do not appear to influence long-term survival. About 35% of lymph node metastases are palpable at the time of presentation, and lateral aberrant thyroid is a subset of these. Because papillary thyroid carcinoma is a well-differentiated tumor, biopsy of a lateral neck mass may appear microscopically as normal thyroid tissue in an abnormal location, thus the term lateral aberrant thyroid. Although lymph node metastasis is not a strong prognostic indicator, the presence of lymph node metastases does influence local recurrence rates. Some authors recommend prophylactic central neck dissection at the time of initial thyroidectomy. Modified neck dissection (sparing the sternocleidomastoid muscle, internal jugular vein, and spinal accessory nerve) is warranted if palpable lymph node metastases are present and clinically are not fixed and at operation are not invading any of these structures. Embryologically, the thyroid descends from the foramen cecum at the base of the tongue to its permanent location in the midline of the neck. Congential anomalies are midline when they occur. A cyst, the most common of which is a thyroglossal duct cyst, may form along this track of descent. However, this is not associated with functional thyroid tissue. Hashimoto’s disease is a benign autoimmune disorder that gradually leads to hypothyroidism. There is no thyroid tissue in the cervical lymph nodes with this disease. By definition, goiter is enlargement of the thyroid itself; it may extend laterally, but there is connection to the central component.

39-Derived from thyroid c-cells: (A) Papillary thyroid cancer (B) Follicular thyroid cancer (C) Medullary thyroid cancer (D) Anaplastic thyroid cancer (E) Follicular variant of papillary thyroid cancer Answer is C Papillary thyroid cancer is the most common thyroid cancer, comprising over 70% of all cases. The second most common type is follicular thyroid cancer. Both papillary and follicular thyroid cancers are derived from thyroid follicular cells and are often grouped together as well-differentiated thyroid cancers. Medullary thyroid cancer is a neuroendocrine tumor derived from thyroid c-cells. Whereas 80% of medullary thyroid cancers are of the sporadic variety, 20% are associated with mutations in the RET proto-oncogene. These familial medullary cancer syndromes include familial medullary thyroid cancer, multiple endocrine neoplasia type 2A (MEN2A), and multiple endocrine neoplasia type 2B (MEN2B). Anaplastic thyroid cancer is the least common and most aggressive thyroid cancer, with a median survival of less than 6 months. Treatment options include external-beam radiation and/or chemotherapy. Follicular variant of papillary thyroid cancer is a subtype of papillary thyroid cancer. Although most thyroid cancers can be diagnosed by fine-needle aspiration (FNA), follicular thyroid cancers cannot. Follicular thyroid cancers have a benign counterpart called follicular adenomas. The only difference between a follicular thyroid cancer and a

Page 22: Q and a Exam for Seniors in general surgery

follicular adenoma is a vascular or capsular invasion, which can only be seen on permanent histology.

40-A 35-year-old woman has a 1-cm infiltrating ductal grade I carcinoma of her right breast diagnosed by excisional biopsy. She is requesting a skin-sparing mastectomy (SSM) and sentinel lymph node biopsy with immediate reconstruction. Which of the following statements is NOT true? (A) The local recurrence rate is higher for this procedure than for a nonskin- sparing mastectomy (B) SSM allows the native skin envelope to be used for immediate reconstruction (C) If she smokes, she is at higher risk for flap necrosis (D) The prior excisional biopsy site must be completely excised to minimize local recurrence (E) Nipple-areolar excision is necessary

Answer is A Skin-sparing mastectomy (SSM) is being offered with increased frequency to patients with early stage breast cancer who are candidates for or desire a mastectomy in lieu of breast conservation. Smaller skin incisions are used around the nipple-areolar complex to preserve the breast envelope of skin. After preoperative planning, the plastic surgeon selects the mode of reconstruction to fill the “envelope” of skin, improving the aesthetic result. Multiple retrospective studies report local recurrence rates ranging from 0 to 7%, which is similar to historical control groups in whom wider skin removal was used. The local recurrence is related to the pathologic characteristics of the breast cancer and the stage of disease, not an inadequate surgical excision. Selection bias is one major limitation to the retrospective studies because most included patients with T1 and T2 tumors, some of whom had more favorable tumor characteristics. In performing the operation, traditional mastectomy landmarks are used as the flaps are elevated (see figure A). The nipple-areolar complex is completely excised in a circular or elliptical fashion (see figure B). To better ensure negative pathologic margins, the prior recent excision biopsy sites should be excised (see figure C). If a sentinel node biopsy is performed, a counter axillary incision is preferred for better exposure. For a positive sentinel node, the completion dissection can be done through the mastectomy or axillary incisions (see figure D). Some surgeons excise core biopsy sites to minimize the potential effect of biopsy track seeding because most of these patients will not be receiving chest wall radiation. Routine excision of core biopsy sites is not universally supported because local recurrence rates are low even without radiation therapy. Tissue flap necrosis is a potential complication and is related to the patient’s history of smoking as well as previous incisions for reduction mammoplasties. Patients with these histories should be cautioned about this potential complication.

Page 23: Q and a Exam for Seniors in general surgery

41-A 58-year-old woman has been in the ICU for 2 weeks because of respiratory failure after an exacerbation of asthma. She is currently intubated for adult respiratory distress syndrome (ARDS) (FiO2, 0.60; positive end-expiratory pressure, 15 cm H2O; static peak compliance, 40 cm H2O) and receiving steroids for the asthma. She is receiving full nutritional support via a small-bore nasointestinal feeding tube when she develops abdominal tenderness, distention, and diarrhea. WBC count is 18,000/mm3. The abdominal computed tomographic showed GIT wall gases. The most appropriate management would be (A) laparotomy (B) colonoscopy (C) laparoscopy (D) upper gastrointestinal contrast study (E) nasogastric decompression and serial examinations Answer is A Pneumatosis intestinalis is an uncommon condition presenting as multiple gas-filled cysts of the gastrointestinal tract. The cysts may be located in the subserosa, submucosa, and, rarely, muscularis layer and vary from microscopic to several centimeters in diameter. They can occur anywhere along the gastrointestinal tract, from the esophagus to the rectum; however, they are most common in the jejunum, followed by the ileocecal region and the colon. Extraintestinal structures such as mesentery, peritoneum, and the falciform ligament may also be involved. There is an equal incidence among males and females, and this condition most commonly occurs in the fourth to seventh decades of life. The cause of pneumatosis intestinalis has not been completely delineated. A number of theories have been proposed, of which the mechanical, mucosal damage, bacterial, and pulmonary theories appear to be the most promising. Pneumatosis may be primary or secondary. About 15% of cases are primary and idiopathic; the cysts are submucosal and usually are limited to the left colon. Secondary pneumatosis comprises 85% of cases and is typically associated with ischemic colitis. Ischemic colitis is a form of nonocclusive ischemic disease usually seen in older patients, without a marked sex predilection. A number of predisposing or precipitating factors have been delineated, including arteriosclerotic heart disease, hypotensive episodes, cardiac and aortic surgery, myocardial infarction, digitalis treatment, arrhythmias, vasculitis, and colonic obstruction. Intestinal ischemia occurs most commonly in the colon. Unlike small bowel ischemia, colonic ischemia is rarely associated with major arterial or venous occlusion. Instead, most colonic ischemia appears to result from low flow and/or small vessel occlusion. Risk factors include vascular disease, diabetes mellitus, vasculitis, and hypotension. In addition, ligation of the inferior mesenteric artery during aortic surgery predisposes to colonic ischemia. Occasionally, thrombosis or embolism may cause ischemia. Although the splenic flexure is the most common site of ischemic colitis, any segment of the colon may be affected. The rectum is relatively spared because of its rich collateral circulation.

Page 24: Q and a Exam for Seniors in general surgery

Patients with ischemic colitis have an abrupt onset of abdominal pain, diarrhea (commonly bloody), and systemic symptoms. The abdomen may be tender diffusely, in a localized area (eg, left lower quadrant), or not at all. In a review of computed tomographic (CT) findings in 54 patients with ischemic colitis, segmental involvement of the colon was seen in 48 patients (89%), with a mean length of involvement of 19 cm (range, 5 to 38 cm). Wall thickness varied between 2 and 20 mm (mean, 8 mm). All parts of the colon were involved. The CT appearance of the colonic wall varied. A wet appearance with heterogeneous areas of edema was seen in 33 patients (61%), a dry appearance with mild homogeneous thickening was seen in 18 patients (33%), and intramural air was present in 3 patients (6%). These authors concluded that CT can be used to confirm the clinical suspicion of ischemic colitis. The clinical course in this patient is highly suggestive of an acute intra-abdominal process, which is confirmed by the finding of pneumatosis intestinalis on the abdominal CT scan. Delayed laparotomy is associated with increased morbidity and mortality; colonoscopy and an upper GI series would yield minimal additional information and delay definitive treatment. Diagnostic laparoscopy may confirm the diagnosis, but has been of limited utility with distended bowel loops and is highly dependent on the experience of the surgeon. It also would not allow assessment of bowel wall edema seen with early mesenteric ischemia.

42-A 25-year-old woman underwent Roux-en-Y gastric bypass and was discharged on postoperative day 2. One month after her operation she began to have intermittent emesis. She presents 6 months later with persistent emesis and now bilateral symmetric thigh numbness and difficulty walking. Workup has revealed no anatomic cause for her emesis. The most likely cause of her neurologic symptoms is a deficiency of (A) iron (B) folate (C) vitamin B1 (thiamine) (D) vitamin B6 (pyridoxine) (E) vitamin B12 (cobalmin) Answer C Roux-en-Y gastric bypass produces weight loss through both restrictive and malabsorptive mechanisms. A small (typically 15 to 30 mL) gastric pouch is formed and connected to a Roux limb of jejunum via a small (10 to 15 mm) anastomosis, restricting food intake. Nutrients travel down the digestive limb and are not effectively absorbed until mixed with biliopancreatic secretions in the common limb. Biliopancreatic diversion with and without duodenal switch combines a more modest restriction of gastric size with a longer biliopancreatic limb, and is more malabsorptive. Nutrient deficiencies are more likely with shorter absorptive limbs and are proportional to weight loss. Mineral and multivitamin supplementation should be routine in patients after bariatric procedures. Mild protein-calorie deficiency is not uncommon, but can become severe and should be suspected with anatomic difficulties such as anastomotic stenosis. Iron deficiency is the most common nutrient deficiency seen after bariatric procedures, with an estimated incidence of 14% to 16% after Roux-en- Y gastric

Page 25: Q and a Exam for Seniors in general surgery

bypass and 21% to 26% after biliopancreatic diversion. Iron deficiency occurs because iron absorption depends on acid exposure to dietary iron and because iron is absorbed mainly in the duodenum and proximal jejunum. Symptoms include microcytic anemia, brittle nails, and headache. Deficiency can occur even in patients taking multivitamins, requiring additional iron supplementation. Vitamin B12 deficiency occurs, particularly after restrictive

after operation. Overall prevalence of vitamin B12 deficiency is estimated at 12% to 33%, but symptoms of deficiency (anemia, glossitis, numbness and tingling of hands and feet, mental status changes) are relatively uncommon. Vitamin B12 levels should be checked 3 to 6 months after bariatric operations and annually or semiannually thereafter. Anemia due to folate deficiency is somewhat less common than vitamin B12 deficiency, and occurs because folate absorption occurs preferentially in the proximal part of the small intestine. Thiamine absorption is in the duodenum and proximal jejunum. Thiamine (B1) deficiency is also known as beriberi, and comes in two forms. Cardiac beriberi presents with cardiac failure, and is not reported in bariatric patients. Most cases of neurologic beriberi after bariatric surgery are related to persistent emesis or hyperemesis. Patients present with symmetric numbess and tingling over the anterior thighs bilaterally progressing to bilateral lower extremity weakness and even paralysis if left untreated. The pathologic mechanism is axonal degeneration. Calcium deficiency is also common after bariatric procedures, again because calcium is preferentially absorbed in the duodenum and proximal jejunum. Vitamin D and other fat-soluble vitamins are also not absorbed well. Vitamin D deficiency aggravates calcium malabsorption. In one report, vitamins A, E, and K were deficient in most patients after biliopancreatic diversion. Vitamin B6 deficiency has not been described after bariatric procedures.

43-Management of uncomplicated appendicitis in a pregnant woman includes all of the following EXCEPT: (A) perioperative fetal monitoring (B) pneumatic compression devices (C) antibiotics and interval appendectomy (D) laparoscopic approach based on surgeon experience and uterine size (E) ultrasonography Answer is C Appendicitis is the most common nonobstetric surgical condition during pregnancy. Symptoms of pain and the location of tenderness vary in pregnancy based on the position of the appendix and its cephalad shift during progressive uterine enlargement. Ultrasonography is a useful diagnostic adjunct to physical examination findings and will also allow for fetal evaluation. The use of ionizing radiation associated with computed tomographic (CT) scan or fluoroscopic imaging should be minimized if possible. Clinical suspicion of appendicitis during pregnancy requires prompt surgical intervention in all trimesters. Nonoperative management using antibiotics and interval appendectomy is no longer warranted

Page 26: Q and a Exam for Seniors in general surgery

in pregnancy. Fetal loss rates of 3% to 5% are observed with acute appendicitis; with perforated appendicitis and progression of intra-abdominal infection, fetal loss rates approach 20%. The choice of open versus laparoscopic appendectomy should be based on surgeon experience and the size of the uterus. The risk of premature labor is greatest during the first postoperative week and mother and fetus should be monitored closely. Pregnancy is a risk factor for DVT and pneumatic compression devices may be appropriate.

44-A 40-year-old man with no co-morbidities presents with diffuse abdominal pain and distention and a 2-day history of nausea and vomiting. He is afebrile and hemodynamically normal, but is anuric and serum creatinine is 3.0 mg/dL. A computed tomographic (CT) scan confirms necrotizing pancreatitis with a large peripancreatic phlegmon. There is no evidence of cholelithiasis or cholecystitis. He has had no prior episodes of pancreatitis. The next step in management should be (A) total parenteral nutrition (TPN) (B) bowel rest, fluid resuscitation (C) surgical pancreatic debridement (D) fine-needle aspiration of peripancreatic fluid (E) prophylactic antifungal agents Answer is B Acute pancreatitis represents a spectrum of disease ranging from a mild, self-limited course to arapidly progressive, severe illness. The mortality rate of severe acute pancreatitis exceeds 20% and some patients diagnosed with mild to moderate acute pancreatitis at the onset of the disease may progress to a severe, life-threatening illness within 2 to 3 days. The most important initial treatment priority is to ensure adequate fluid resuscitation to maintain organ perfusion, to maintain pancreatic perfusion and limit the degree of necrosis, and to improve major organ perfusion in an effort to prevent multiple organ failure. A 2004 international consensus conference developed recommendations for the management of the critically ill patient with severe acute pancreatitis, defined as acute pancreatitis with organ dysfunction. Recommendations included no routine use of prophylactic systemic antibacterial or antifungal agents in patients with necrotizing pancreatitis; no pancreatic debridement or drainage for sterile necrosis; limitation of debridement or drainage to those with infected pancreatic necrosis and/or abscess confirmed by radiologic evidence of gas or results of fine-needle aspirate of peripancreatic fluid; and delay of operative necrosectomy and/or drainage at least 2 to 3 weeks to allow for demarcation of the necrotic pancreas. These guidelines are similar to previous guidelines developed by the International Association of Pancreatology and more recent guidelines published by the Japanese. The potential role of prophylactic antibiotics for severe acute pancreatitis remains controversial, and further studies are required to provide adequate data and further define whether antibiotics have any role in this challenging patient population. The 2003 Cochrane systematic review, which included 4 studies, demonstrated an advantage for antibiotic therapy for survival (OR 0.32, p=0.02)

Page 27: Q and a Exam for Seniors in general surgery

and reduction of pancreatic sepsis (infected necrosis, OR 0.51, p=0.04). Most of these studies were limited by small sample size and lack of double blinding. A recent double-blind placebo-controlled multicenter study demonstrated no benefit of antibiotic prophylaxis with respect to the risk of development of infected pancreatic necrosis. Enteral nutrition is superior to parenteral nutrition in patients with severe acute pancreatitis.

45-A 91-year-old man presents with small bowel obstruction. Gallstone ileus is suspected. After resuscitation, the best treatment approach would be: (A) conservative management with nasogastric suction (B) laparotomy and milking of the stone into the large bowel (C) cholecystectomy, repair of the small bowel component of the fistula, common bile duct exploration, and extraction of the stone through an enterotomy (D) removal of the stone through a proximal enterotomy (E) colonoscopic disruption and extraction of the stone Answer is D Gallstone ileus represents only 1% of all cases of small bowel obstruction, but accounts for 15% of complete obstruction in patients over age 70. These stones are large (diameter > 2.0 cm) and cannot be milked through the ileocecal valve. If the stone can be dislodged proximally, an enterotomy is performed and the stone is removed. Because these stones often cause irritation and ulceration, they may be impacted and therefore not amenable to proximal dislodgement. In these cases, a limited small bowel resection of the segment containing the stone is appropriate. The stone is most often lodged within the terminal ileum and is beyond the reach of a colonoscope. Colonoscopy and stone extraction have been described in a few cases in which the gallstone traversed the ileocecal valve and caused large bowel obstruction.. Removal of the gallbladder is not necessarily indicated during the first operation unless biliary symptoms are present. The small bowel component of the cholecystenteric fistula can be repaired at the time of cholecystectomy; cholecystectomy and fistula repair can be performed as a delayed operation. Addressing only the small bowel obstruction is acceptable if the patient is unstable. A common bile duct exploration is not necessary because the gallstone usually travels through the fistula between the gallbladder and intestine. Simple enterotomy should be the procedure of choice for patients with gallstone ileus. The one-stage procedure including cholestectomy and fistula repair should be reserved for highly selected patients with absolute indications.

46-A 45-year-old woman with active Crohn’s proctitis has a symptomatic anterior transphincteric anovaginal fistula. The most appropriate management would be: (A) a draining seton (B) saucerization

Page 28: Q and a Exam for Seniors in general surgery

(C) fibrin glue (D) a cutting seton (E) an endorectal advancement flap Answer is A Perianal fistulas are a common manifestation of Crohn’s disease. Multiple therapeutic options are available. Factors to be considered include associated disease activity and anatomic relations of the fistula, including level of internal opening, associated tracts, and the amount of sphincter traversed by the fistula. Perianal Crohn’s disease is a chronic relapsing condition that places the anal sphincter at lifelong risk of injury, and with disease-related diarrhea places these patients at high risk for potential fecal incontinence. All efforts should be made at sphincter-preserving treatments. Fistulotomy should be reserved for short, superficial fistulas. Surgical repairs such as saucerization or placement of a cutting seton will not heal in the presence of grossly active proctitis and may result in debilitating, slow-healing wounds. Fistula symptoms are often promulgated by areas of undrained infection. Drainage of the abscess may be followed by a loose draining seton and medical therapy in hopes of resolving the active inflammation in preparation for definitive surgical repair. In the setting of minimal or no proctitis, an endorectal advancement flap has been successful in 50% to 70% of patients with Crohn’s fistulas. Fibrin glue repair is an appealing healing option in that it is a simple and repeatable sphincter-preserving technique; however, it has a high failure rate, especially in fistulas that are short and/or associated with Crohn’s disease.

47-The distal margin of resection for sigmoid diverticulitis should be: (A) 2 cm distal to the colon inflammation (B) the sacral promontory (C) uninflamed distal sigmoid (D) the peritoneal reflection (E) normal rectum Answer E Diverticular disease is a common acquired anatomical condition of the colon rarely occurring before age 30. The incidence increases with age, and this condition is present in 75% of Americans over age 80. Diverticulitis is a peridiverticular infection resulting from perforation of a diverticulum and subsequent leakage of stool from the colon. The process usually resolves with a short course of antibiotics. Surgical resection of the inflamed segment of colon is indicated for patients with complicated diverticulitis as manifested by generalized peritonitis, abscess formation, fistula, or medical intractability. Operation for uncomplicated diverticulitis is generally reserved for patients who have had multiple recurrences. Surgical goals include resection of the diverticulitis with margins of uninvolved colon in each side of the inflamed segment of bowel. The distal margin of resection for sigmoid diverticulitis should be normal rectum because the rectum does not have any tinea coli and the incidence of rectal diverticulitis is exceedingly rare. The risk of recurrent diverticulitis is considerably lower when the distal margin of resection is to the rectum instead of the distal sigmoid colon.

Page 29: Q and a Exam for Seniors in general surgery

48-The use of computed tomography (CT) in suspected appendicitis: (A) lowers the incidence of negative appendectomy (B) has become more accurate over the past 10 years (C) eliminates the need for serial re-examinations (D) has decreased the rate of perforated appendicitis at operation (E) may detect pelvic abnormalities in women Answer is E Although computed tomography (CT) is now more commonly used for suspected appendicitis, its diagnostic value remains questionable. In a study comparing 100 consecutive cases in 2000 with 100 consecutive cases in 1994, the use of CT had increased fourfold, but the negative appendectomy rate had actually increased from 12% to 17%, although this was not significant. The only significant effect found was an increase in the length of stay in the emergency department. Accuracy of the CT was 80% in 2000 and 81% in 1994. The rate of perforated appendicitis was 13% in 2000 and 11% in 1994. Technical improvements in CT do not appear to have increased its accuracy. Re-examination remains valuable for suspected appendicitis. In a study of CT (and ultrasound) in 100 consecutive patients in which CT was followed by re-examination, CT and re-examination resulted in a change in diagnosis in one third of cases, and the diagnosis of pelvic inflammatory disease or ruptured ovarian cyst could be made in 6 women. The negative appendectomy rate was 17%, which is consistent with many other studies in which CT was not used. No study has compared CT alone with re-examination alone, and ethical considerations would make such a study unlikely.

49-A 66-year-old otherwise healthy man with epigastric abdominal pain and anemia is evaluated with upper endoscopy. Four 1-cm submucosal tumors are identified in the body of the stomach. Biopsies show carcinoid tumor. Fasting serum gastrin concentration is 950 pg/mL (normal, 100 pg/mL). Further work-up establishes a diagnosis of gastric carcinoid tumor associated with gastric achlorhydria, pernicious anemia, and associated hypergastrinemia. The best management would be (A) total gastrectomy (B) proton pump inhibitors (C) antrectomy with gastroduodenostomy (D) proximal gastrectomy with esophagogastrostomy (E) enucleation of all 4 tumors Answer is C Gastric carcinoid tumors occur in three distinct settings, and because treatment depends on the etiology, work-up must be directed at determining the underlying etiology. Some patients have sporadic tumors, which usually occur as larger, solitary tumors. A second type, present in this patient, is associated with gastric achlorhydria, pernicious anemia, and associated hypergastrinemia. These

Page 30: Q and a Exam for Seniors in general surgery

patients tend to have multiple small gastric carcinoids in the body of the stomach caused by the trophic effect of gastrin on intestinal mucosa. Finally, some gastric carcinoids occur in the setting of Zollinger- Ellison syndrome (ZES) and hypergastrinemia. Because atrophic gastritis is much more common than ZES, the most cost-effective test to determine the cause of hypergastrinemia in the patient described in this scenario is one that confirms atrophic gastritis, not ZES. Octreotide scans would be indicated only for patients with ZES or large sporadic tumors. Low serum levels of vitamin B12, histologic confirmation of atrophic parietal cell mucosa, and high gastric pH all confirm the diagnosis and/or rule out ZES. Serum levels of chromogranin A should be measured in all carcinoid patients. Baseline and follow-up levels can be used to monitor these patients. By definition, serum gastrin levels are normal in the sporadic carcinoid group. Sporadic gastric carcinoid tumors are potentially lethal, and partial gastrectomies and lymphadenectomies are indicated for these patients. Multiple small tumors in the setting of hypergastrinemia have low (but not zero) malignant potential. Unless the tumors are larger than 1 to 2 cm, treatment should focus on eliminating hypergastrinemia. Antrectomy (resection of all gastrin-producting cells) and normalization of serum gastrin levels will result in regression of all tumors less than 1 cm in most patients with atrophic gastritis, making enucleation or resection of multiple small gastric carcinoids unnecessary. Proximal gastrectomy adds morbidity and does not address the etiology of the disease, and total gastrectomy is unnecessarily radical treatment. Antisecretory agents such as proton pump inhibitors or histamine H2 receptor antagonists have no role in treatment of these patients.

50-Which of the following statements about the treatment of hemorrhoids is TRUE? (A) Surgical hemorrhoidectomy is appropriate first-line therapy for grade III or grade IV hemorrhoids (B) Rubber band ligation, sclerotherapy, and infrared coagulation are equivalent options for in-office treatment of hemorrhoids (C) Similar postprocedure pain occurs after stapled hemorrhoidectomy and rubber band ligation (D) Excision or incision and clot evacuation are equally effective for treatment of thrombosed external hemorrhoids (E) Approximately 20% of patients will experience pain due to thrombosed external hemorrhoids after rubber band ligation Answer is A Five percent of the population complains of hemorrhoids. Hemorrhoids above the dentate line are designated as internal hemorrhoids and classified by grade, as shown in the table. Rubber band ligation is the most effective office procedure for symptomatic internal hemorrhoids, and has a lower rate of recurrence than sclerotherapy or infrared coagulation. Complication rates with rubber band ligation are low and thrombosis of external hemorrhoids secondary to rubber band ligation is uncommon. When compared with rubber band ligation, stapled hemorrhoidectomy resulted in more postoperative pain for grade III internal hemorrhoids. For patients with thrombosed external hemorrhoids, those who have been symptomatic for less than 48 to 72 hours can be treated with surgical

Page 31: Q and a Exam for Seniors in general surgery

excision of the thrombosed hemorrhoid. This is superior to simple incision and clot evacuation because the lesion typically consists of multiple small intravascular thromboses rather than a single hematoma. Surgical hemorrhoidectomy is appropriate for primary treatment of grade IV and large grade III hemorrhoids. Primary surgical hemorrhoidectomy is also appropriate for combined internal and external hemorrhoids with significant prolapse.

51-Which of the following statements about anal fissures is NOT true? (A) Nonoperative treatment with increased fluid intake, fiber ingestion, and stool softeners is the best initial management (B) Topical nitrates are only marginally effective at healing anal fissures (C) Topical calcium channel blockers are more effective than oral calcium channel blockers at healing anal fissures (D) Recurrence is common after initial successful treatment with botulinum toxin (E) Lateral internal sphincterotomy should be recommended only after topical nitrates, calcium channel blockers, or botulinum therapy has failed. Answer is E An anal fissure is a longitudinal tear in the squamous lining of the anal canal between the anal verge and the dentate line. Fissures typically result in severe anal pain that is often excruciating during defecation. Patients may also experience some degree of bleeding. In most cases the fissure is caused by an episode of constipation with passage of a hard stool. Diagnosis is made by history and confirmed by examination of the anal canal. Because most will heal spontaneously, acute anal fissures are best treated with conservative measures consisting of stool softeners, fiber supplements, and increased fluid intake. Topical nitrates such as 0.2% nitroglycerin ointment can help to lessen the pain of anal fissures, but are marginally associated with a healing rate superior to placebo. The principal side effect of topical nitrates is headache, which can limit its effectiveness. Topical calcium channel blockers can also promote healing, and have fewer side effects than nitrates. The availability of topical calcium channel blockers may be limited, however, and oral calcium channel blockers are not as effective as topical application. Injection of botulinum toxin into the internal sphincter allows for healing of up to 80% of chronic anal fissures, but recurrence is common. Due to the relatively low healing rates and high recurrence rates with topical treatments, surgical sphincterotomy remains an accepted alternative to pharmacologic treatment when conservative treatment with stool softeners, fiber supplements, and increased fluid intake has not been successful. Operation may be appropriately offered to such patients without a trial of pharmacologic therapy.

52-A 56-year-old woman has rectal cancer found on colonoscopy performed for evaluation of rectal bleeding. The patient’s primary concern is whether operation will result in a permanent colostomy. Which of the following is the primary determinant of the feasibility of avoiding a permanent colostomy? (A) Distance of the tumor from the proximal anal canal

Page 32: Q and a Exam for Seniors in general surgery

(B) Presence of enlarged lymph nodes on computed tomographic (CT) scan (C) Distance of tumor from dentate line (D) Distance of tumor from anal verge (E) Extension of tumor into posterior vaginal wall or bladder. Answer is A The objectives for surgical management of rectal cancer are the achievement of adequate surgical margins, and appropriate lymph node dissection. Total mesorectal excision is the current standard to achieve adequate radial margins and an optimal lymph node dissection. The appropriate length of the distal margin of resection is a subject of controversy. Most surgeons would agree that a 2-cm distal margin with wide lateral dissection is sufficient; however, some clinical evidence suggests a 1-cm margin may be equally effective for attaining local control and improving survival. For an appropriate oncologic resection while still preserving the sphincters (to avoid a permanent stoma), the distal resection margin cannot encompass the anal sphincter mechanism. If the distal resection margin extends beyond the superior margin of the anal canal, and thus into the sphincter mechanisms, then an abdominal perineal resection is required. The distance of the tumor from the top of the sphincter mechanism or the proximal anal canal is critical in detecting whether an adequate cancer resection can be performed without excising the anal sphincters. The length of the anal canal and the relative position of the dentate line vary from person to person and are less reliable landmarks. Presence of lymph node metastasis should not affect the decision to create a permanent stoma. Local tumor involvement of the posterior vaginal wall or bladder involvement does not necessarily eliminate the ability to perform a coloanal anastomosis.

53-A previously healthy 58-year-old man with lower gastrointestinal hemorrhage is hemodynamically stable. Gastric lavage returns clear bilious fluid. Colonoscopy demonstrates continued brisk bleeding, but no definite bleeding site is identified. Which of the following statements is TRUE?

A. Angiographic embolization is associated with an 80% success rate and infrequent ischemic complications

B. The use of heparin or urokinase may increase the sensitivity of angiography with minimal increase in complication rates

C. Vasopressin infusion has similar efficacy and fewer ischemic complications compared with angiographic embolization.

D. Tagged RBC scan should be performed routinely prior to angiography E. Vascular ectasias account for 30% of massive lower gastrointestinal

hemorrhages Answer is A Angiographic embolization is a relatively new modality used for the treatment of severe colonic hemorrhage. Multiple reports indicate an 80% success rate of stopping the hemorrhage with this procedure. Ischemic complications after angiographic embolization are rare when superselective techniques are used. Heparin or urokinase can increase the sensitivity of angiography for identifying

Page 33: Q and a Exam for Seniors in general surgery

the site of bleeding, but is associated with a 20% complication rate and should be used only in highly selected patients. Angiographic embolization is as effective as vasopressin infusion in controlling lower gastrointestinal hemorrhage and avoids the vasoconstrictive side effects of vasopressin that can occur in 10% of cases. Tagged RBC scans can be useful in identifying patients who are likely to be actively bleeding and thus would warrant angiography, but lack adequate accuracy in locating the site of bleeding. When patients such as the one described in this clinical scenario are known to have active brisk bleeding, then angiography should be performed without the bleeding scan. The most common causes of massive lower gastrointestinal bleed are diverticular disease and inflammatory bowel disease. Although arteriovenous malformations or vascular ectasias have been extensively reported, their incidence as a cause of massive lower gastrointestinal hemorrhage is 1% to 3%.

54-Endoscopic therapy for treatment of Barrett’s esophagus with low-grade dysplasia includes all of the following EXCEPT:

A. endoscopic plication B. multipolar electrocoagulation C. endoscopic mucosal resection D. photodynamic therapy E. argon beam coagulation

answer is A Barrett’s esophagus (BE) is a premalignant condition in which normal squamous epithelium in the distal esophagus is replaced by intestinal metaplasia. Patients with BE have a 0.5% per year risk of developing adenocarcinoma of the esophagus. Treatment options for BE include medical therapy, operation, endoscopic ablative techniques, and endoscopic mucosal resection. Proton pump inhibitors and Nissen fundoplication are used for treatment of Barrett’s without dysplasia. Once dysplasia or cancer is identified, endoscopic ablation/resection or esophagectomy is the primary treatment modality. Multipolar electrocoagulation (MPEC) relies on current passing between electrodes and achieves a depth of damage of 1.7 to 4.8 mm. Strictures are associated with this technique in 1% of cases. Argon beam plasma coagulation (APC), another endoscopic ablation technique, transfers electrical energy by means of the ionized gas argon to achieve very high temperatures and desiccation. APC’s depth of damage is 1 to 3 mm. Perforations and deaths have resulted. Photodynamic therapy (PDT) uses systemic administration of 5-aminolevulinic acid (5-ALA), a photosensitizing agent that acts selectively on rapidly proliferating or malignant cells. Laser light of the appropriate wavelength exposed to the photosensitized cells causes a cytotoxic reaction mediated by oxygen. Perforations are rare, but strictures can occur in as many as 36% of cases. The depth of damage is 2 to 6 mm. PDT in combination with omeprazole has been statistically shown to decrease the progression of cancer from 28% to 13% in patients with high-grade dysplasia. Cryoablation and YAG lasers have also been used as ablative techniques. Endoscopic mucosal resection is performed by piecemeal resection of Barrett’s segments under direct visualization. For macroscopically visible lesions, endoscopic resection as opposed to ablation allows analysis of the actual pathologic specimen. However, complete resection of areas not visible or those

Page 34: Q and a Exam for Seniors in general surgery

that are multifocal and long is often difficult. Possible complications include strictures (3% to 17%) and perforations (2% to 9%). The choice of technique continues to be controversial and randomized trials are rare. Current guidelines recommend esophagectomy in medically fit patients with high-grade dysplasia, reserving endoscopic treatments for poor-risk patients and for those involved in clinical trials. Endoscopic plication is currently being used for treatment of gastroesophageal reflux disease (GERD), not Barrett’s.

55-Which of the following statements about nonoperative treatment of clinically diagnosed appendiceal phlegmon is TRUE?

A. Interval appendectomy should be scheduled within 6 weeks of discharge B. Interval appendectomy is probably not necessary C. Recurrence of acute appendicitis is over 50% D. Recurrence and complication rate is higher if the patient initially had

peritonitis or abscess formation E. Length of hospital stay is longer for patients with recurrence than it is for

interval appendectomy. Answer is B In a study of nearly 33,000 patients with appendicitis, 3% (1012 patients) did not have an appendectomy. Of these 1012 patients, 85% (864) did not have an interval appendectomy. With a median follow-up of 4 years, only 5% of these 864 patients had a recurrence, and one third of this group were successfully managed nonoperatively and none had a second recurrence. Patients in this large study who presented initially with abscess or peritonitis did not have any higher incidence of recurrence. Further, length of hospital stay for interval appendectomy was not only longer than it was for a recurrence, but also longer than it was for patients who were treated by emergency appendectomy on the first admission.

56-All of the following are contraindications to neostigmine therapy for suspected colonic pseudoobstruction EXCEPT:

A. bradycardia B. history of bronchospasm C. renal failure D. abdominal pain E. mechanical bowel obstruction

answer is D Randomized controlled trials support the use of intravenous neostigmine as a treatment for acute colonic pseudo-obstruction. Clinical response is defined as a prompt evacuation of flatus and/or stool with reduction in abdominal distention. Patients with underlying bradyarrhythmias or those receiving - adrenergic antagonists may be more susceptible to neostigmine-induced bradycardia. Similarly, neostimine increases airway secretions and bronchial reactivity, which

Page 35: Q and a Exam for Seniors in general surgery

may exacerbate bronchospasm. Because renal excretion accounts for 50% of drug clearance, patients with renal impairment may have an increased or prolonged vagomimetic response after treatment. Mechanical obstruction should be excluded as a contributing factor prior to administration of neostigmine. This is most easily accomplished with a retrograde single-contrast water-soluble enema, documenting a lack of proximal distention/distal collapse. If obstruction cannot be excluded, colonoscopic decompression is preferable to neostigmine. Although the presence of abdominal pain in association with colonic distention would be a concern, pain is not a contraindication in the absence of other signs and symptoms suggestive of intestinal ischemia or perforation.

57-A 43-year-old woman has gallstone pancreatitis that resolves in 2 days with conservative treatment. She has no abdominal complaints and her liver and pancreatic laboratory values have returned to normal. She is scheduled for laparoscopic cholecystectomy. Which of the following statements is TRUE?

A. Intraoperative cholangiography is associated with a decreased risk of biliary tract injury

B. The procedure should be scheduled for 6 weeks after resolution of symptoms

C. Intraoperative cholangiography in this patient will identify choledocholithiasis in 50% of cases

D. Preoperative endoscopic retrograde cholangiopancreatography (ERCP) should be performed

E. The sensitivity of magnetic resonance cholangiopancreatography (MRCP) for choledocholithiasis in this patient is less than 50%

Answer is A Up to 25% of patients with gallstone pancreatitis will have a recurrent attack within 30 days if they do not undergo cholecystectomy. Recurrence may take the form of biliary colic, cholecystitis, choledocholithiasis, or pancreatitis, and is not related to the severity of the initial presentation or evidence of choledocholithiasis. Although most patients will not be harmed by waiting up to 30 days, recurrence may be seen as early as 2 weeks after initial presentation. The decision to perform routine versus selective intraoperative cholangiography during the operative procedure centers on two potentially preventable outcomes—retained common duct stones and injury to the extrahepatic biliary system. Common duct stones in patients with gallstone pancreatitis can be identified by MRCP, ERCP, or intraoperative cholangiography. MRCP is 80% to 100% sensitive in detecting stones > 2 mm and can be used to identify retained stones, but an additional procedure may then be required to perform stone extraction, making the cost-benefit of routine MRCP questionable. Routine preoperative ERCP will subject 70% to 80% of patients to an unnecessary procedure, potentially increasing cost and complications. If ERCP is to be used in an algorithm to manage patients with gallstone pancreatitis, selective postoperative ERCP minimizes exposure to an unnecessary procedure. Selection may be based on symptoms, laboratory evaluation, or results of intraoperative cholangiography. Routine intraoperative cholangiography is performed to identify patients with choledocholithiasis, although the incidence of retained stones is not decreased

Page 36: Q and a Exam for Seniors in general surgery

when routine intraoperative cholangiography is used in either the open or laparoscopic approach. This is related to both cannulation failure rate (reported at 5% to 10% for laparoscopic cholangiography) and misinterpretation of the cholangiogram obtained. However, routine intraoperative cholangiography performed during laparoscopic cholecystectomy in patients with cholecystitis, pancreatitis, or jaundice can decrease the incidence of injury to the common bile duct by identifying unclear anatomy. This benefit of intraoperative cholangiography in complex cases is also seen in open cholecystectomy. It is difficult to show that routine intraoperative cholangiography is beneficial in reducing biliary tract injury in uncomplicated cases because biliary duct injury occurs infrequently.

58-The standard for location of mesh implantation for open ventral hernia repair is:

A. superficial to the anterior rectus sheath with at least 4 cm overlap B. between the anterior rectus sheath and the anterior rectus muscle with at

least 4 cm overlap C. sutured to the fascial edge of the defect D. deep to the posterior rectus sheath/transversalis fascia with at least 4 cm

overlap E. mesh reinforcement of fascial margins with linear closure

answer is D Several different techniques have been used for implantation of mesh into a ventral hernia defect. The goal of the repair is to implant the mesh to establish a tension-free repair, using the mechanics of the abdominal wall to distribute pressure on the mesh over as large an area as possible. If the mesh is placed anterior to the anterior fascia, even with 2 cm of overlap, there is still pressure applied to the mesh only through the hernia defect itself, which increases the amount of pressure on a smaller area of mesh. This similar principle applies to placement of the mesh anterior to the rectus muscle and also sutured to the fascial edge. Posteriorly placed mesh is the only repair that distributes pressure over a larger area of mesh. Mesh in this position has been shown to be associated with the lowest recurrence rate, and can be placed by either an open or laparoscopic approach. Mesh with barrier properties to prevent adherence to abdominal viscera should be used.

59- Nine months after right inguinal hernia repair, a 65-year-old man presents with chronic right groin pain. The operative report describes routine tension-free herniorrhaphy with mesh, with the mesh placed in the anterior position. Physical examination and computed tomographic (CT) scan do not reveal a recurrent hernia. He has reproducible pain with percussion just medial to his right anterior superior iliac spine. The most appropriate step now would be:

A. magnetic resonance imaging (MRI) of the lumbar spine B. groin exploration C. mesh excision and native tissue hernia repair

Page 37: Q and a Exam for Seniors in general surgery

D. ilioinguinal and iliohypogastric nerve blocks E. neurontin

Answer is D

Mesh is almost universally accepted as the repair of choice for inguinal hernia, and complications from mesh placement are becoming more frequent. Early complications include wound infection and acute hernia recurrence. Late complications include hernia recurrence, chronic pain from the mesh implantation itself, and injury to one of the nerves that bring sensation to the groin and medial thigh. The ilioinguinal and iliohypogastric nerve are at risk with an anterior approach, with the ilioinguinal nerve being involved most frequently. This patient’s physical findings do not indicate a need for lumbar spine MRI. Neurontin is used for groin pain after hernia repair, but would not be indicated at this time. For patients with chronic groin pain after hernia repair, ruling out an occult hernia as an etiology is important. Physical examination and computed tomographic (CT) scanning are both reasonable options to make this evaluation. In the absence of any evidence of hernia on either of these examinations, blind groin exploration is not indicated. To complete the patient’s work-up for postoperative hernia pain, the ilioinguinal and iliohypogastric nerves should be blocked to see whether the patient’s pain is relieved. This evaluation looks for a nerve entrapment syndrome. If injection relieves the pain, then ilioinguinal and/or iliohypogastric neurectomy would be indicated. Chronic pain from the mesh implantation itself tends not to be reproduced with percussion over the ilioinguinal nerve. It is a more dull chronic pain that exists in the inguinal floor itself. Excision of the mesh and repair of the hernia with autogenous tissue is indicated in a patient with such a syndrome who presents 9 months after a hernia repair with mesh. Referral to a chronic pain clinic would not be appropriate.

60- A 52-year-old diabetic woman with Child’s C cirrhosis undergoes repair of an incarcerated umbilical hernia. At operation, a defect containing infarcted omentum is identified and repaired. On postoperative day 1, the patient develops upper gastrointestinal bleeding from esophageal varices and requires 4 units of packed RBCs. On the fifth postoperative day, her abdominal wound is painful and erythematous. All of the following are significant potential risk factors for developing this surgical site infection EXCEPT:

A. intraoperative hyperglycemia B. perioperative hypoxemia C. postoperative transfusion D. cirrhosis E. age

answer E Surgical site infections remain a major source of postoperative morbidity, prolonging hospital stays and increasing total surgical costs. Although appropriately timed antibiotic prophylaxis (administered within 1 hour prior to incision and discontinued within 24 hours or after 1 or 2 doses) effectively decreases infectious wound complications, several inherent and modifiable risk factors have also been identified that dramatically affect the incidence of surgical

Page 38: Q and a Exam for Seniors in general surgery

site infections. Multivariate analyses have identified variables independent of preoperative factors (such as evidence of chronic illnesses, particularly cirrhosis, malnutrition, cardiac, respiratory and/or renal failure, and the presence of a preoperative urinary catheter) that more than double the incidence of surgical site infections postoperatively. Low partial pressure of oxygen in contaminated tissue has been hypothesized to define an ideal infectious environment. Numerous studies have documented that postoperative blood transfusion is associated with an increase in surgical site infections, but the effect may be due to postoperative anemia and poor tissue oxygen delivery. Maintenance of perioperative normothermia, strict glycemic control, and avoidance of perioperative hypoxemia are independent practice factors that can be manipulated to decrease the chance of developing a surgical site infection. Although age leads to a progressive reduction in immunity and has been identified as a risk factor for global postoperative infectious complications, age is not a risk factor for surgical site infection or deep parietal infections.

61- A recently married, otherwise healthy 34-year-old man presents with symptomatic bilateral hernias. The left-sided hernia extends into the scrotum. The right side is recurrent after repair 20 years previously. He is interested in starting a family, and is concerned that operation may affect fertility. Preoperative sperm analysis is normal. He should be advised that

A. bilateral tension-free mesh repair (anterior, open approach) is the procedure of choice

B. totally extraperitoneal (TEP) laparoscopic mesh repair eliminates risk of injury to vas deferens and spermatic vessels

C. mesh repairs have been implicated as a cause of obstructive azoöspermia D. Bassini repairs will minimize the potential for infertility E. his risk of ischemic orchitis and testicular atrophy is < 1%

answer is C Despite more than a century of modern approaches to the treatment of groin hernias, a consensus has yet to be reached on the optimal technique of repair. Variables to consider include open vs laparoscopic, tension-free mesh vs tissue repair, and anterior vs preperitoneal vs intraperitoneal approach. Most surgeons in North America have adopted some type of prosthetic mesh repair as their operation of choice for primary hernias in adults. However, in Canada and elsewhere, the Shouldice repair, incorporating multiple layers of tissue instead of mesh, has many faithful adherents. Almost all mesh repairs use nonabsorbable materials, and may be a single sheet, a composite (PTFE and polypropylene), a plug, or a device incorporating mesh attached to a pre-formed flexible ring. Commonly recognized complications include infection, hematoma/hemorrhage, nerve entrapment, recurrence, and ischemic orchitis. Less frequently cited is the potential for infertility. A recent article implicating polypropylene mesh in the development of azoöspermia, presumably secondary to an intense fibrotic reaction around the vas deferens, has raised concerns. Ischemic orchitis is an uncommon result in straightforward primary hernias, but the risk increases from less than 1% to more than 5% with recurrent and/or large scrotal hernias such as a sliding hernia with sigmoid colon as a component. In the former, meticulous attention to the spermatic vessels, which may have been compromised by

Page 39: Q and a Exam for Seniors in general surgery

previous dissection, is essential. In the latter, attempts to resect the distal sac (beyond the pubic tubercle) in large scrotal hernias significantly increase the risk of testicular devascularization. There is no evidence that a Bassini repair reduces the incidence of fertility. “Lipoma” of the spermatic cord is probably a misnomer. The lesion is actually herniation of preperitoneal fat through the lateral aspect of the internal ring. It is easily mistaken for a hernia on physical examination, can cause pain, and when large may sufficiently dilate the ring to predispose to visceral herniation. Lipomas may be missed at laparoscopic repair, and the subsequent persistent groin mass has been interpreted as a recurrence, leading to re-exploration. Most experienced hernia surgeons recommend their removal, which can be done safely without increasing morbidity.

62-Which of the following is an absolute contraindication for liver transplantation? (A) Autoimmune hepatitis (B) Hepatitis C infection (C) Synchronous extrahepatic malignancy (D) Hepatitis B infection (E) Fulminant hepatic failure Answer is C Absolute contraindications:

1) -advanced , uncorrectable cardiac or pulmonary disease 2) -severe irreversible pulmonary HTN 3) -hypotension on vasopressor 4) -recent intracranial hrg 5) -irreversible neurological impairement 6) -HIV 7) -uncontrolled sepsis 8) -extrahepatic malignancy 9) -inability to comply with posttransplant regimen.

63- Liver transplantation is contraindicated for a:

A. 56-year-old man 4 days after orthotopic liver transplant who has acute hepatic artery thrombosis and for whom arterial thrombectomy was unsuccessful

B. 48-year-old man with a history of alcoholic cirrhosis who has been abstinent for 5 years

C. 68-year-old woman with nonalcoholic cirrhosis related to fatty liver disease D. 52-year-old man with cirrhosis who has just undergone a wedge resection

of a 5-mm focus of hepatocellular carcinoma involving the middle lobe of the right lung

E. 32-year-old woman in the ICU with variceal bleeding who has portal hypertension due to hepatitis C-induced cirrhosis

Answer is D

Page 40: Q and a Exam for Seniors in general surgery

Liver transplantation is the standard treatment for end-stage liver disease, and 1-year survival rates of 85% to 90% are typical at most centers. The number of liver transplants performed annually from 1998 to 2002 in the US increased slightly from 4516 to 5329, but 6444 transplants were performed in 2005, reflecting increased educational efforts about organ donation. Hepatitis C infection is currently the most common condition resulting in liver transplantation. Patients with a history of alcoholism or alcohol abuse are considered for transplantation after careful assessment by a psychiatrist or addiction specialist. The length of abstinence required for listing varies. When comparing patients who receive transplants for alcoholic cirrhosis and nonalcoholic liver disease, there was no difference in survival. Patients older than 60 are undergoing transplantation with increasing frequency. Although seniors with far-advanced end-stage liver disease are high-risk for liver transplantation when compared with younger patients, most seniors who survive liver transplantation have full or only minimally limited functional status, and advanced age alone should not exclude a patient from liver transplant. The incidence of hepatic artery thrombosis (HAT) after liver transplantation ranges from 2% to 9% in various series, but HAT can be a devastating problem that requires re-transplantation in nearly half the patients due to biliary complications or deteriorating liver graft function. When arterial revascularization cannot be successfully accomplished in a patient with early HAT, mechanisms organized by the United Network of Organ Sharing (UNOS) facilitate early re-transplantation. All patients with cirrhosis are at higher risk for developing hepatocellular carcinoma, ~1% to 4% risk per year in patients with hepatitis C. Liver transplantation can be safely performed in patients with small tumors (< 5 cm) in the setting of cirrhosis. The presence of extrahepatic disease is a contraindication to proceeding with liver transplantation.

64- Neonatal Hirschsprung’s disease is associated with all of the following EXCEPT: (A) failure to pass meconium in the first 24 hours of life (B) dilated proximal bowel on prenatal ultrasound study (C) feeding intolerance (D) higher incidence with trisomy 21 (E) male predominance Answer B Hirschsprung’s disease is defined as a congenital absence of ganglion cells in the myenteric and submucosal plexuses of the intestinal tract. The aganglionic segment involves the distal rectum and extends proximally for variable distance, most commonly ending in the sigmoid colon. Aganglionosis causes muscular contraction of the involved segment, causing the normal proximal bowel to become dilated secondary to a functional obstruction. Hirschsprung’s disease is typically diagnosed during the first few days of life. Clinically, the disease is characterized by feeding difficulty, abdominal distention, obstructive plain films despite a patent gastrointestinal tract, and failure to pass meconium within the first 24 hours of life due to the distal aganglionic segment of bowel. There is a notable male predominance and a higher incidence of Hirschsprung’s disease in children with trisomy 21.

Page 41: Q and a Exam for Seniors in general surgery

Diagnosis is generally made by contrast enema demonstrating a transition point between the contracted distal bowel and the normal dilated proximal intestine. A transanal rectal suction biopsy of the distal rectal mucosa and submucosa can be performed at the bedside, and the absence of submucosal ganglion cells with nerve fiber hypertrophy will confirm the diagnosis. Fetal ultrasound study has not proven useful in the prenatal diagnosis of Hirschsprung’s disease. Dilated proximal bowel on prenatal ultrasonography is consistent with imperforate anus, intestinal atresia, or meconium ileus secondary to cystic fibrosis.

65- A 6-week-old infant presents with failure to thrive, dehydration, and progressive postprandial non-bilious emesis. He has a palpable mid-epigastric mass. The most likely diagnosis is:

A. duodenal atresia B. annular pancreas C. intestinal duplication cyst D. neuroblastoma E. hypertrophic pyloric stenosis

Answer is E Progressive postprandial nonbilious emesis, failure to thrive, dehydration, and a palpable midepigastric mass in a 1- to 2-month-old infant is characteristic of hypertrophic pyloric stenosis. In this setting, the only further diagnostic work-up required is serum electrolyte determination, because hypochloremic, hypokalemic, metabolic alkalosis is common and requires adequate fluid resuscitation with saline prior to pyloromyotomy. The pyloromyotomy may be performed either open or via a laparoscopic approach. Duodenal atresia and annular pancreas typically present with feeding intolerance in the first few hours to days of life. A hallmark of duodenal atresia is a double-bubble sign on plain abdominal film, reflecting gaseous dilatation of the stomach and proximal duodenum. Both duodenal atresia and annular pancreas are treated by duodenoduodenostomy. Intestinal duplications are rare and typically difficult to palpate given the propensity to be cystic. Neuroblastomas generally arise in the adrenal gland or paravertebral sympathetic chain ganglia and may present as a palpable abdominal mass; however, the tumors do not typically cause symptoms of gastric outlet obstruction.

66- A laparoscopic cholecystectomy is performed in a 55-year-old woman. Pathology reveals gallbladder adenocarcinoma confined to the mucosal lamina propria. Appropriate management includes: (A) observation and surveillance follow-up only (B) excision of gallbladder fossa and laparoscopic port sites (C) excision of gallbladder fossa, port sites, portal lymphadenectomy (D) excision of gallbladder fossa, port sites, portal lymphadenectomy, excision of common bile duct (E) irradiation of the gallbladder bed

Page 42: Q and a Exam for Seniors in general surgery

Answer is A Long-term survival in patients with gallbladder carcinoma is generally limited to those with isolated, incidentally discovered disease confined to the mucosa. In the setting of an incidentally discovered gallbladder carcinoma confined to the mucosal lamina propria (stage T1a) following laparoscopic cholecystectomy, no further surgical treatment is required. Patients must be followed-up closely for the potential development of port site metastases or locoregional recurrence. Five-year survival rates range from 83% to 100%. For tumors invading the muscularis of the gallbladder (T1b), most authors recommend consideration of port site excision and extended cholecystectomy with wedge hepatectomy of the gallbladder fossa and regional lymphadenectomy. For T2 lesions invading perimuscular connective tissue not beyond serosa or liver, excision of the medial hepatic segments with regional lymphadenectomy with or without excision of the extrahepatic bile duct is recommended. This offers a five-year survival rate of 60% to 80%. Lesions that extend beyond the serosa of the gallbladder (T3, T4) represent advanced disease. In the absence of extensive nodal disease or distant metastases, extended surgical resection offers the only possibility of long-term survival. Radiation treatment is not warranted for this T1 lesion.

67- Principles of treatment of gastrointestinal stromal tumors (GIST) include all of the following EXCEPT:

a. segmental resection with negative margins b. resection of involved tissues including adjacent organs c. en bloc lymphadenectomy d. tyrosine kinase inhibitor (imatinib, Gleevec) in patients with

residual disease e. multitargeted tyrosine kinase inhibitor (Sunitinib) in patients

refractory or intolerant to Gleevec answer is C The goal of treatment of all gastrointestinal stromal tumors (GIST) is complete gross resection with an intact pseudocapsule. Although they may appear ominous on CT, GISTs often project extraluminally from the stomach or small intestine and displace rather than invade adjacent organs. They can often be lifted away from surrounding structures, although en bloc resection is necessary in some cases because of dense adhesions. Segmental resection of the stomach or intestine should be performed with the goal of achieving negative resection margins. Wider resection of uninvolved tissue is of no additional benefit. Routine lymphadenectomy is unnecessary because nodal metastases are rare. Prognosis is influenced by the completeness of resection. Prior to 2000, there was no known effective single-agent or combination chemotherapy regimen for unresectable or metastatic GIST. Recent data indicate that molecularly targeted therapy with the tyrosine kinase inhibitor imitanib (Gleevec) can produce dramatic, rapid, clinical benefit for GIST patients with advanced or unresectable disease. The malignant cells in most GISTs overexpress the product of the KIT proto-oncogene, a tyrosine kinase called c-Kit, which can be detected by

Page 43: Q and a Exam for Seniors in general surgery

immunohistochemistry. A smaller subset are c-Kit negative, but have mutations in a second tyrosine kinase receptor, the platelet-derived growth factor receptor-alpha (PDGFRa). Imitanib is effective in tumors with the mutant isoforms of KIT as well as some GISTs that contain mutant isoforms of PDGFRa. Emerging data suggest that selection of clones with secondary mutations in c-Kit is the most common mechanism of imitanib resistance. Possible methods of overcoming imitanib resistance include the use of more powerful selective tyrosine kinase inhibitors of c-Kit together with selective inhibitors of downstream pathways. Early reports suggest efficacy for the multitargeted tyrosine kinase inhibitor, Sunitinib. This agent is not yet commercially available in the United States.

68- Oral fluoropyrimidine is: (A) Fluorouracil (5-FU) (B) Capecitabine (C) Cetuximab (D) Irinotecan (E) Bevacizumab Answer is B Fluorouracil (5-FU) has been the standard cytotoxic chemotherapeutic agent for treatment of stage IV colon cancer for almost 50 years. In more recent decades, randomized multi-institutional studies have confirmed its benefit for patients with stage III disease and even for those with stage II disease. Although surgical resection cures 50% to 60% of patients with stage III disease, up to 20% more can be cured if 5-FU is administered postoperatively. More recently developed agents for treatment of stage IV colorectal cancer show great promise for decreasing risks or side effects of therapy, being easier to administer, or acting via a different mechanism. Capecitabine (Xeloda) is an oral fluoropyrimadine prodrug that is converted through a multistep activation to fluorouracil at the cellular level. Its oral form makes it easier to administer and its 45-minute half-life allows it to be given twice daily. Controlled studies have demonstrated that its effectiveness is at least equal to that of 5-FU plus leucovorin with somewhat less neutropenia. Xeloda is associated with a significant incidence of severe hand-foot syndrome (palmar-plantar erythrodysesthesia). In the US, multi-agent therapy has become standard treatment. 5-FU is often combined with either oxaliplatin or irinotecan, an inhibitor of topoisomerase I that was initially introduced as monotherapy for cancers resistant to 5-FU but has also demonstrated improved results in combination with 5-FU, a regimen known as FOLFIRI. The cisplatinum derivative oxaliplatin has become first-line treatment for patients with stage IV colorectal cancer when used in combination with 5-FU and leucovorin (FOLFOX regimen). FOLFOX and FOLFIRI regimens are comparable in their overall efficacy and tolerability. Two additional agents provide targeted molecular treatment for advanced colon cancer:

Page 44: Q and a Exam for Seniors in general surgery

bevacizumab (Avastin) and cetuximab (Erbitux). Avastin is a monoclonal antibody directed against vascular endothelial growth factor (VEGF) and acts by inhibiting the growth of tumor blood vessels. Its addition to 5-FU increases the response rate, median time to disease progression, and median duration of survival. Erbitux is a monoclonal antibody that blocks the signaling by HER-1/EGFR (endothelial growth factor) receptors that stimulate tumor cell proliferation (migration, adhesions, and angiogenesis and inhibit apoptosis). Erbitux is not sufficient by itself for tumor cytotoxicity, but blocking HER1-EGFR signaling is believed to render the tumor cells more vulnerable to cytotoxic chemotherapeutic agents. Although the optimal combination of dosing and sequencing of agents is not fully defined, these combination agents have already improved median survival for patients with metastatic colorectal cancer from 12 months to 2 years.

69- A 44-year-old woman with a history of seizure disorder pulls a pan of hot grease onto herself during a seizure. She has burns to her chin, chest, and abdomen. On initial evaluation, her wounds are red with pale areas centrally, slow to blanch, and dry in the central portion of the wound. This patient has most likely sustained a: (A) superficial burn (B) superficial partial-thickness burn (C) deep partial-thickness burn (D) full-thickness burn (E) first-degree burn Answer is C The superficial or first-degree burn usually has a damaged but intact epidermis that is red in appearance. The damaged epidermis may exfoliate in approximately 7 to 10 days, but is usually healed underneath at the time of separation. A sunburn is a superficial or first-degree burn. A partial-thickness burn means that damage has occurred to the dermis. These burns can be further subdivided into superficial or deep partial-thickness burns. The superficial partial-thickness burn extends into the papillary dermis. The skin appendages in the dermis are still viable and the hair follicles are difficult to remove. This is characterized by blister formation with moist and pink wounds beneath. These burn wounds are extremely painful and blanch easily when compressed. In a favorable environment, these wounds will re-epithelialize within 2 weeks by migration of the epithelial cells lining the hair follicles and sebaceous glands onto the surface of the dermis to replace the damaged and destroyed keratinocytes. Deep partial-thickness burns extend into the reticular dermis. The skin appendages are damaged and the hair follicles usually are disrupted. These burns can be mottled and dry in appearance and will likely form an eschar. The eschar is the product of coagulated proteins from the injured skin. The wounds have slow capillary refill because of damage to the dermal plexus and hemagglutination in subjacent capillaries. These wounds may heal by re-epithelialization from the dermal appendages and wound periphery after 3 weeks and will most likely leave a hypertrophic scar if treated nonsurgically. Full-thickness burn injuries involve the entire thickness of the burn. No dermal appendages are viable for re-epithelialization. These wounds contain necrotic tissue and the eschar is usually thick, dry, and white or charred in appearance. These wounds have a leathery feel and are not supple like nonburned skin. These wounds may even be dark red in

Page 45: Q and a Exam for Seniors in general surgery

appearance as a result of dermal vascular stasis, but they do not blanch with pressure and are generally insensate. Surgical intervention and excision and grafting are necessary unless the lesions are very small and can be left to separate and heal by constant contraction and epithelialization.

70- Sentinel lymph node biopsy for melanoma

a. predicts prognosis b. improves survival c. is indicated for melanomas < 1 mm thick d. is associated with a complication rate of 23% e. should be followed by completion lymph node dissection in patients

who have positive results Answer is A The likelihood of detecting metastatic deposits in the sentinel lymph node depends on tumor thickness: approximately 1% if the thickness of the tumor is less than 0.8 mm, 8% if it is 0.8 to less than 1.5 mm, 23% if it is 1.5 to less than 4.0 mm, and 36% if it is 4.0 mm or greater. Because many large retrospective studies have shown a strong negative correlation between the presence of metastatic melanoma in sentinel nodes and survival, sentinel lymph node biopsy (SLNB) is currently the most powerful staging and prognostic tool. Performance of this minimally invasive procedure provides prognostic information and identifies candidates for systemic adjuvant treatment. Early results from 2120 patients in the Sunbelt Melanoma Trial estimate complication rates of 4% for SLNB and 23% for SLNB with completion lymph node dissection. Because patients with melanomas that are 1.0 mm or less in thickness rarely have nodal disease, SLNB is not commonly performed but could be considered if pathologic examination showed negative prognostic features such as ulceration or Clark level IV to V invasion. Further studies are needed to determine the additional benefit of completion lymph node dissection in patients who have positive results on sentinel node biopsy.

71- Which of the following statements IS TRUE concerning the role of glutamine in

total parenteral nutrition?

A. Glutamine is an essential amino acid. B. Glutamine appears not to be of primary benefit in critical illness. C. Glutamine is not included in most standard TPN solutions. D. Glutamine is the primary energy source for mucosal cells of the small

bowel and colon.

Answer: C

Glutamine is the most studied gut-specific nutrient. Glutamine has been classified

as a nonessential or nutritionally dispensable amino acid since glutamine can be

synthesized in adequate quantities from other amino acids and precursors.

Glutamine is not included in most nutritional formulas and has been eliminated

from TPN solutions because of its relative instability and short half life compared

to other amino acids. With few exceptions, glutamine is present in oral enteral diets

Page 46: Q and a Exam for Seniors in general surgery

but only at relatively low levels characteristic of the concentration in most animal

and plant stores (about 7% of total amino acids). Several recent studies, however,

have demonstrated that glutamine may be an essential amino acid during critical

illness, particularly as it relates to supporting the metabolic requirements of the

intestinal mucosa. These studies demonstrate that dietary glutamine is not required

during states of health but appears to be beneficial when glutamine depletion is

severe and/or when intestinal mucosa is damaged by insults such as chemotherapy

or radiation therapy. The addition of glutamine to enteral diet reduces the incidence

of gut translocation but these improvements are dependent upon the amount of

supplemental glutamine and the type of insult studied. Glutamine-enriched TPN

partially attenuates villous atrophy that develops during parenteral nutrition. The

use of intravenous glutamine in patients appears to be safe and effective in its

ability to maintain muscle glutamine stores and improve nitrogen balance. In

contrast to glutamine, short chain fatty acids are primary energy source for

colonocytes.

72- Which of the following statements concerning bile duct strictures due to chronic pancreatitis is FALSE?

A. Most patients present with progressive jaundice. B. Strictures are classically long and tapered involving the entire

intrapancreatic bile duct. C. Patients may be asymptomatic and diagnosed only by persistent elevation

of serum alkaline phosphatase. D. An excellent option for surgical management is choledochoduodenostomy.

Answer: A

The clinical presentation of patients with common bile duct strictures secondary to

chronic pancreatitis is variable. A large number of patients are asymptomatic with

a diagnosis of bile duct strictures suggested only by abnormal liver function tests.

The serum alkaline phosphatase appears to be the most sensitive laboratory finding

and is elevated in over 80% of patients. Although in most cases, transient jaundice

may occur, progressive jaundice is rare. Cholangiography will demonstrate a

classic long smooth gradual tapering of the common bile duct throughout its entire

intrapancreatic segment. Biliary reconstruction is the appropriate management for

most patients. Choledochoduodenostomy is preferred by many surgeons because

it does not divert bile from the duodenum, is technically easier to perform, and

leaves the jejunum intact for any associated procedures that may be required for

decompression of the obstructed gastrointestinal tract or pancreatic duct.

73- Abnormalities of the sphincter of Oddi have been recently recognized to cause symptoms which are referable to the biliary tree or pancreas. Which of The following statements IS FALSE concerning sphincter of Oddi motor function.

Page 47: Q and a Exam for Seniors in general surgery

A. The sphincter’s basal resting pressure is 10 to 15 mm Hg above duodenal pressure

B. Contraction of the sphincter occurs with CCK stimulation. C. Vagal stimulation results in relaxation of the sphincter. D. Manometry of the sphincter of Oddi may be performed at the time of

ERCP to characterize basal pressure, amplitude, frequency of contraction, and direction of propagation of contractile waves.

E. Stenosis of the sphincter of Oddi is characterized by abnormally elevated basal pressure on sphincter of Oddi manometrics.

Answer: B

The sphincter of Oddi is about 4 to 6 mm in length. The sphincter’s basal resting

pressure is about 13 mm Hg above duodenal pressure. The sphincter exhibits

phasic contractions at a frequency of 4 per minute and a duration of 8 seconds. The

regulation of bile flow is primarily controlled by the sphincter and not by the

surrounding smooth muscle of the duodenum. Relaxation of the sphincter occurs

with CCK stimulation leading to diminished amplitude of phasic contractions and

reduced basal pressure, allowing increased passive flow of bile into the duodenum.

Parasympathetic stimulation also causes intermittent relaxation of the sphincter,

and sympathetic splanchnic stimulation causes increased pressure. Abnormalities

of the sphincter of Oddi may cause symptoms which are referable to the biliary tree

or pancreas. Manometry of the sphincter of Oddi may be performed at the time of

ERCP to characterize its basal pressure, the amplitude and frequency of

contractions, and the direction of propagation of contractile waves. Stenosis of the

sphincter of Oddi is characterized by abnormally elevated basal pressure (> 40 mm

Hg) whereas dyskinesia is characterized by abnormalities of other manometric

parameters.

74-Which of the following veins is preserved in performing the extensive

esophagogastric devascularization procedure described by Sugiura?

A. Left gastric (coronary) vein.

B. Short gastric vein.

C. Splenic vein.

D. Left gastroepiploic vein.

Answer: A

The Sugiura procedure consists of devascularization of the esophagus to the

inferior pulmonary vein and the proximal two thirds of the stomach, splenectomy,

and distal esophageal transection. The devascularization component should be

done as close to the esophagus and stomach as possible. The coronary vein and

paraesophageal collaterals are preserved to maintain an effective portal-systemic

collateral pathway and thereby discourage reformation of varices.

75- Which of the following statements is true concerning the pathophysiology of

small bowel obstruction?

Page 48: Q and a Exam for Seniors in general surgery

A. Most of gas seen on plane abdominal radiographs is produced by gas forming microorganisms

B. Intestinal blood flow initially increases to the bowel wall in early bowel obstruction

C. In the face of obstruction, myoelectrical activity of the bowel is consistently increased

D. Constipation is an early symptom.

Answer: B

When a loop of bowel is obstructed, intestinal gas and fluid accumulate.

Approximately 80% of the gas seen on plane abdominal radiographs is attributable

to swallowed air. In the setting of acute pain and anxiety, patients with intestinal

obstruction may swallow excessive amounts of air. Fluid accumulates

intraluminally with open-or closed-loop small intestinal obstruction due to a

number of factors. Experimental studies and clinical investigations demonstrate

that elevation of luminal pressures above 20 cm H2O inhibits absorption and

stimulates secretion of salt and water into the lumen proximal to an obstruction. In

closed-loop obstruction, luminal pressures may exceed 50 cm H2O and may

account for a substantial proportion of a luminal fluid accumulation. In simple,

open-loop obstruction, distention of the lumen by gas rarely leads to a luminal

pressure higher than 8–12 cm H2O. Thus, in open-loop obstruction, the

contributions of high luminal pressures to hypersecretion may not be important. In

response to heightened luminal pressure, total blood flow to the bowel may initially

increase. Subsequently, however, blood flow to the bowel is compromised as

luminal pressures increase, bacteria invade, and inflammation leads to edema

within the bowel wall. Accumulation of gas and fluid in the obstructed lumen also

leads to changes in myoelectrical function in the gut, proximal and distal to the

obstructed segment. In response to distension, the obstructed segment itself may

dilate, a process known as “receptive relaxation.” At sites proximal and distal to

the obstruction, changes in myoelectrical activity are time-dependent. Initially,

there may be intense periods of activity and peristalsis. Subsequently,

myoelectrical activity is diminished and interdigestive migrating myoelectrical

complex (MMC) is replaced by ineffectual and seemingly disorganized clusters of

contractions.

76- A 25-year-old man is involved in an automobile accident with resultant injuries

including bilateral closed femur fractures, left pulmonary contusion, and closed

head injury. On post-injury day 4, significant upper gastrointestinal hemorrhage

begins. Endoscopic examination reveals an area of confluent ulceration with

bleeding in the gastric fundus. Endoscopic hemostasis fails. Appropriate

immediate management includes:

A. Lavage of gastric contents with iced saline B. Urgent total gastrectomy C. Selective arterial infusion of vasopressin via the left gastric artery D. Insertion of Sangstaken-Blakemore balloon

Page 49: Q and a Exam for Seniors in general surgery

Answer: c

Initial efforts to control gastric hemorrhage consist of gastric lavage using warmed

saline. Lavage serves to fragment existing clots and to remove any pooled blood,

reducing fibrinolysis at bleeding sites. Over 80% of patients who present with upper

gastrointestinal hemorrhage stop bleeding using this approach. Definitive

treatment of ongoing acute active stress bleeding by antacids is largely

unsuccessful. Administration of H2-receptor blocking agents once active

gastrointestinal bleeding has commenced is also usually ineffective as a definitive

form of therapy.

The endoscope has become the preferred therapeutic as well as diagnostic

instrument with electrocautery and laser photocoagulation capabilities. If

endoscopic therapy fails, angiography offers an additional means for the control of

bleeding by selective infusion of vasopressin into the splanchnic circulation via the

left gastric artery. Vasopressin is administered by continuous infusion through the

catheter at a rate of 0.2 to 0.4 IU/min for a maximum of 48 to 72 hours.

About 10% to 20% of patients with acute stress ulcers continue to bleed or have

recurrent bleeding despite these measures. In these patients, total gastrectomy has

a mortality ranging from 17% to 100%. In general, operative mortality rates for acute

stress-induced hemorrhage range from 30% to 60% regardless of the surgical

procedure undertaken.

77- Which of the following is most reliable for confirming the occurrence of a

significant esophageal caustic injury?

A. History of the event.

B. Physical examination of the patient.

C. Barium esophagraphy.

D. Endoscopy.

Answer: D

In the absence of physical or radiographic evidence of upper airway obstruction

or esophagogastric perforation, the presence of a significant caustic injury can be

defined reliably only by direct visualization at the time of endoscopy. Although the

history may shed light on the possibility of a burn and its severity, all too often the

event goes unwitnessed or the type and amount of ingested substance are not

known with certainty. The identification of oropharyngeal burns clearly indicates

the need for endoscopy, but as many as 70% of patients with such lesions escape

associated esophageal injury. Conversely, 10% to 30% of patients with no external

evidence of burns have subsequently been confirmed by esophagoscopy to have

sustained damage. In the absence of an identifiable perforation, a barium

esophagogram can rarely be considered unequivocally diagnostic of acute injury,

though such an injury may be suggested when the esophagus appears atonic and

dilated, rigid and persistently narrowed, or excessively irritable. Because of the

importance of early confirmation of the presence or absence of a significant

Page 50: Q and a Exam for Seniors in general surgery

esophageal burn as a guide to formulating appropriate treatment, esophagoscopy

should be performed expeditiously as soon as sufficient time has elapsed to allow

gastric emptying and stabilization of the patient, preferably within the first 12 to 48

hours after ingestion.

78- In which of the following clinical situations peritoneal lavage is NOT indicated?

A. A patient with suspected intraabdominal injury who will undergo prolonged general anesthesia for another injury outside the abdomen.

B. A patient with a high velocity abdominal gunshot wound. C. A patient with an abdominal knife wound. D. A patient with major noncontiguous injuries (i.e., chest and lower extremity).

Answer: B

Peritoneal lavage is a standard technique to detect significant intraabdominal

hemorrhage after blunt trauma. Its applicability after low-velocity gunshot or stab

wounds is less clear, but it has no place in the evaluation of high-velocity gunshot

wounds. Abdominal paracentesis can be used in place of peritoneal lavage when

the suspicion of intraabdominal hemorrhage is high and time is critical. Specific

indications for peritoneal lavage and blunt trauma include a number of conditions

such as a patient with major noncontiguous injuries, a patient with suspected

intraabdominal injury in whom physical examination is unreliable or impossible due

to the need for prolonged general anesthesia for another injury. Peritoneal lavage

is not useful for patients with abdominal gunshot wounds; all of these patients

require immediate laparotomy. When local examination of a stab wound suggests

penetration to the anterior fascia and peritoneum, diagnostic peritoneal lavage may

help discriminate between those with significant and insignificant injuries.

79- Weaning patients from maximum ventilator support usually involves:

A. Weaning PEEP first, tidal volume second, and the fraction of inspired oxygen

(FIO 2) third.

B. Weaning FIO 2 first, ventilator rate second, and PEEP third.

C. Weaning FIO2 first, PEEP second, and tidal volume third.

D. Weaning FIO 2 first, PEEP second, and ventilator rate third.

Answer: D

When the inspired oxygen concentration is greater than 60% for more than 24 to 28

hours, the risk of oxygen toxicity increases. PEEP is usually weaned to physiologic

levels (i.e., 5 to 7 cm. H 2O) before weaning either rate or tidal volumes. Generally,

the optimal tidal volume to achieve alveolar recruitment is selected and usually is

not decreased unless peak airway pressures increase. If decreases in ventilatory

rate are not tolerated, airway pressure support can be added.

Page 51: Q and a Exam for Seniors in general surgery

80- A 45-year-old woman presents with a weeping eczematoid lesion of her nipple. Which of the following statements IS TRUE concerning her diagnosis and management?

A. Treatment is with warm compresses and oral antibiotics. B. Biopsy of the nipple revealing malignant cells within the milk ducts is

invariably associated with an underlying invasive carcinoma, but not in situ ductal carcinoma.

C. The appropriate treatment is mastectomy. D. The lesion always represents a high-risk disease with a significant risk of

subsequent metastatic disease.

Answer: c

Paget’s disease is characterized by weeping, eczematoid lesion of the nipple. There

is often accompanying edema and inflammation. Biopsy of the nipple reveals

malignant cells within the milk ducts. The lesion is invariably associated with an

underlying invasive or in situ ductal carcinoma. The prognosis of Paget’s disease

is that of the underlying cancer. Standard treatment is mastectomy with axillary

lymph node dissection only if invasive cancer is present.

81-Chylous ascites is the accumulation of chyle within the peritoneal cavity. ALL

of the following statements are true concerning chylous ascites EXCEPT?

A. The cisterna chyli lies at the anterior surface of the third and fourth lumbar

vertebrae and receives lymphatic fluid from the mesenteric lymphatics.

B. Chylous ascites is most commonly associated with abdominal lymphoma.

C. Paracentesis and analysis of chylous fluid typically reveals elevated

triglycerides, protein, and leukocyte levels, with a predominance of

lymphocytes.

D. The mortality rate in adults with chylous ascites is in excess of 21%.

Answer: A

Chylous ascites is accumulation within the peritoneal cavity of chyle, a lymphatic

fluid with a high lipid content. Access of intestinal lipids to the circulation is via

mesenteric lymphatics that enter the cisterna chyle, which in turn becomes the

thoracic duct which eventually enters the venous system at the junction of the left

subclavian and internal jugular veins. The cisterna chyli lies at the anterior

surface of the first and second lumbar vertebrae slightly to the right of the aorta.

Chylous ascites may result from injury to major lymphatic duct or the cisterna.

However for lymphatic leakage to persist, widespread occlusion of

lymphaticovenous collaterals within the abdomen must be present. Malignancy is

the predominant cause (88%) of spontaneous chylous ascites in adults, with

lymphoma the most common malignancy. Diagnostic studies must include not

only documentation of lymphatic origin of the abdominal fluid but also an attempt

to delineate the cause of chylous ascites.

Page 52: Q and a Exam for Seniors in general surgery

Paracentesis and analysis of chylous fluid typically reveals elevated triglycerides,

protein, and leukocyte levels, with a predominance of lymphocytes. Unfortunately,

cytology is seldom positive despite the presence of malignancy.

Lymphangiography may define the site of lymphatic leak for patients in whom the

leak is from the cisterna or retroperitoneal lymphatics but not when from the

mesenteric or hepatic lymphatics. Of noninvasive studies, CT is the test of choice,

with a high diagnostic yield in nontraumatic chylous ascites in adults. Frequently,

laparotomy with node biopsy is required for histology and typing in cases

suspected to be cancer, particularly for lymphoma.

Treatments for chylous ascites have been directed toward decreasing lymph and

triglyceride accumulation. Successful resolution of chylous ascites has been

achieved using a fat-restricted diet with added medium-chain triglycerides in an

attempt to reduce lymphatic transport of triglycerides and perhaps intestinal

lymph flow. Although there have been reports of success using such dietary

manipulation, many failures have been reported. Therefore, in most patients with

chylous ascites, treatment is likely to be successful only when directed toward the

underlying cause. For patients with lymphoma, therapy effective against

lymphoma is likely to eliminate chylous ascites.

The prognosis for patients with chylous ascites is much better in infants and

children than in adults, principally because of the differences in causes of the

condition. A mortality of 21% is reported in infants and children whereas a

mortality of 88% has been noted in adults. Patients with chylous ascites with

associated neoplasms typically have the gravest prognosis.

82-The following statements is true concerning the epidemiology of inguinal

hernias.

A. Inguinal hernias occur with a male-to-female ratio of about 3:1

B. Femoral and umbilical hernias are more common in women, with a female-

to-male ratio of 4:1

C. The frequency of inguinal hernias increases with age

D. Almost all umbilical hernias occur in the pediatric age group.

Answer: C

Inguinal hernias are the most frequently occurring hernia by a factor of five over

other individual types. Umbilical hernias constitute about 14% of hernias, femoral

hernias about 5%, and other types are rare. There is a male prevalence in inguinal

hernias of about 7:1 (male-to-female), whereas there is a female dominance in

femoral and umbilical hernias of 8:1 and 7:1 (female-to-male), respectively. For

inguinal hernia, which occurs at all age levels, frequency increases with age.

Umbilical hernias have a bimodal distribution, peaking in the pediatric population

and then in the 40 to 60 year group, in which the hernias are principally

paraumbilical.

Page 53: Q and a Exam for Seniors in general surgery

83-which of The following statements is true concerning umbilical hernias in

adults.

A. Most umbilical hernias in adults are the result of a congenital defect carried

into adulthood

B. A paraumbilical hernia typically occurs in multiparous females

C. The presence of ascites is absolute contraindication to elective umbilical

hernia repair.

D. Incarceration is uncommon with umbilical hernias.

Answer: B

An umbilical hernia in a child is usually considered to be congenital. Only about

10% of umbilical hernias in adults are thought to be the result of a congenital

defect carried into adulthood. Most adult umbilical hernias are acquired and are

called paraumbilical hernias. The paraumbilical hernia typically occurs in a

multiparous female. Other patients with increased intraabdominal pressure,

particularly with concomitant chronic abdominal distension as from ascites, are

also at increased risk for the development of paraumbilical hernias. Umbilical and

paraumbilical hernias vary from small to extremely large. Incarceration is frequent

in the large hernias, which typically have a small neck.

Indications for umbilical hernia repair in adults include symptoms, incarceration,

large hernia relative to the neck, and trophic changes in the overlying skin.

Among adults with associated ascites, repair is advocated to avoid potentially

serious complications. The presence of discoloration or ulceration of overlying

skin or a rapid increase in size of the hernia herald impending rupture.

Spontaneous rupture of the hernia in these patients can be catastrophic and is

frequently associated with mortality rates approaching 30%. By comparison,

elective umbilical hernia repair can be performed safely in patients with ascites

with acceptable morbidity and mortality.

84-Retroperitoneal fibrosis is a fibrosing condition of retroperitoneum, which is of

significance as it generally encompasses the ureters and eventually causes

hydronephrosis and kidney damage. Which of the following statements is true

concerning this condition?

A. The majority of cases are of a known cause.

B. A history of use of methysergide for treatment of migraine headaches

would be significant

C. There is no known association of malignancy with retroperitoneal fibrosis

D. The disease occurs more commonly in women than in men.

Answer: B

Retroperitoneal fibrosis is a rare condition in which fibrosis develops in the

retroperitoneal space. The ureters frequently will become encompassed by the

process eventually causing hydronephrosis and kidney damage. Retroperitoneal

fibrosis occurs most commonly in the fifth and sixth decades with a 2:1 male-

Page 54: Q and a Exam for Seniors in general surgery

female predominance. The pathophysiology of retroperitoneal fibrosis remains to

be delineated. In fully two-thirds of cases, retroperitoneal fibrosis is idiopathic,

however, an autoimmune process has been suggested as a potential cause. About

12% of cases of retroperitoneal fibrosis have been associated with the use of

methysergide, a serotonin agonist used for vascular and migraine headache, and

in this subgroup females outnumber males 2:1. Primary or metastatic malignancy

in the retroperitoneum is found in 8% of patients with retroperitoneal fibrosis.

Sarcomas are the most common primary tumors, but non-Hodgkin and Hodgkin

lymphomas and ureteral cancer have also been found. Metastases have originated

from cancer of the stomach, breast, colon, carcinoid, pancreas, prostate, ovary,

and cervix. The focus of tumor may be small but may induce desmoplasia that is

grossly indistinguishable from benign variance of retroperitoneal fibrosis.

85-ALL of the following structures are derived from the external oblique muscle

and its aponeurosis EXCEPT?

A. The inguinal or Poupart’s ligament

B. The lacunar ligament

C. The superficial inguinal ring

D. The conjoined tendon.

Answer: D

The external oblique muscle and its aponeurosis, with its inferiorly and medially-

directed fascicles and the overlying innominate fascia lie deep to the

subcutaneous tissue. The inguinal ligament (Poupart’s ligament) is the inferior

edge of the external oblique aponeurosis and extends from the anterior superior

iliac spine to the pubic tubercle, turning under itself posteriorly and then

superiorly to form a shelving edge. Medially, the inguinal ligament turns under

even further to form the lacunar ligament, as part of its insertion on the pubis. The

superficial inguinal ring is a triangular opening in the external aponeurosis, with

its apex superiorly in position slightly above and lateral to the pubic tubercle,

through which the cord exits the inguinal canal. The conjoined tendon is

commonly alluded to in descriptions of inguinal hernia repairs. The conjoined

tendon is the fusion of the aponeurosis of the internal oblique and transversus

abdominis muscles.

86-Once a diagnosis of acute appendicitis has been made and appendectomy

decided upon, which of the following is true?

A. Prophylactic antibiotics should not be administered.

B. Prophylactic antibiotics are not necessary unless there is evidence of

perforation.

C. If the appendix is not ruptured and not gangrenous, antibiotics may be

discontinued after 24 hours.

D. Multiple antibiotics are in all cases preferable to a single agent.

Answer: C

Page 55: Q and a Exam for Seniors in general surgery

It is generally held that patients with a diagnosis of acute appendicitis should

receive antibiotics such as cefoxitin or cefotetan. Administration can be

discontinued after 24 hours if the appendix is not gangrenous or ruptured.

Multiple antibiotics are unnecessary in straightforward cases.

87-The most diagnostic imaging method of acalcular acute cholecystitis is:

A. CT of the abdomen.

B. Ultrasonography of the gallbladder.

C. Oral cholecystogram.

D. Radionuclide (HIDA) scan of the gallbladder

Answer D

88-A female patient is seen in the emergency room with reproducible right lower

quadrant tenderness. The approximate incidence of finding a normal appendix on

right lower quadrant exploration in similar non-selected patients is which of the

following:

A. 2-5% B. 5-10% C. 22-26% D. 30-40%

Answer: c

Appendectomy is the most common surgical procedure performed on an

emergency basis in Western medicine. Appendicitis has a negative appendectomy

rate of approximately 22% to 26% in broad based reviews. The perforation rate is

as low as 3.6% in a subset of young males, although this rises substantially when

the children or the elderly are included. Likewise, young females represent a

group at particularly high risk for other intraabdominal pathology.

89-ALL of the following cause visceral pain from the abdominal organs EXCEPT?

A. Stretching and contraction

B. Traction, compression, torsion

C. Cutting

D. Certain chemicals.

Answer: C

Abdominal pain can be divided into three categories; visceral, somatic, and

referred. The intramural sensory receptors of the abdominal organs are

responsible for visceral pain. A diverse group of destructive stimuli to the

abdominal viscera are painless. For example, almost all abdominal organs are

insensitive to pinching, burning, stabbing, cutting, and electrical and thermal

stimulation. The same is true for the application of acid and alkali to normal

mucosa.

Page 56: Q and a Exam for Seniors in general surgery

The general classes of visceral stimulation that result in abdominal pain include:

(1) stretching and contraction; (2) traction, compression, and torsion; (3) stretch

alone; and (4) certain chemicals. Mediating receptors for these responses are

located intramurally in hollow organs, on serosal structures such as the visceral

peritoneum and capsule of solid organs, within the mesentery and the mucosa.

These receptors are polymodal, or responsive to both mechanical and chemical

stimuli. Mucosal receptors respond primarily to chemical stimulation. Visceral

pain almost always heralds intra-abdominal disease but may not indicate the need

for surgical therapy. When visceral pain is superceded by somatic pain, the need

for surgical intervation becomes likely.

90-Prospective studies have shown incidental appendectomy to be advantageous

in which of the following patient groups?

A. Children undergoing staging laparotomy for malignancy who are then

planned for chemotherapy.

B. HIV infected patients.

C. Patients over 60 years of age.

D. Patients with spinal cord injuries.

E. None of the above.

F. All of the above.

Answer: E

Several studies have looked at incidental appendectomies in a variety of

populations. The deficiency in all past studies of this issue is the lack of

prospective long-term trials to assess the true cost and benefit.

Incidental appendectomy is clearly not indicated in the elderly and in patients

undergoing laparatomy for staging of Hodgkin’s disease. These two specific

groups have been shown to have increased perioperative risks with incidental

appendectomy. No prospective studies have addressed the issue of HIV infected

or spinal cord injured patients. While incidental appendectomies may be

performed safely in general, it is difficult to justify any increase in operative risk

without demonstrable benefit.

-----------------------------------------------------------------------------------------------------------

91-Visceral pain is typically:

A. Well localized

B. Sharp

C. Mediated via spinal nerves

D. Perceived to be in the midline.

Answer: D

Peritoneum is a continuous visceral and parietal layer. The nerve supply to each

layer is separate. The visceral layer, i.e., the layer surrounding all intraabdominal

Page 57: Q and a Exam for Seniors in general surgery

organs, is supplied by autonomic nerves (sympathetic and parasympathetic) and

the parietal peritoneum is supplied by somatic innervation (spinal nerves). The

pathways relaying the sensation of pain differ for each layer and differ in quality

as well. Visceral pain is characteristically dull, crampy, deep, aching and may

involve sweating and nausea. Parietal pain is sharp, severe and persistent.

Visceral organs have very little pain sensation, but stretching of the mesentery

and stimulation of the parietal peritoneum cause severe pain.

Normal embryologic development of the abdominal viscera proceeds with bilateral

midline autonomic innervation that results in visceral pain usually being

perceived as arising from the midline. Epigastric pain is typical of foregut origin.

Periumbilical pain signifies pain emanating from the midgut. Hypogastric or lower

abdominal midline pain indicates a hindgut origin.

92-A 26-year old woman in her first trimester of pregnancy presents with a 2-day

history of right lower quadrant pain and fever. Physical examination reveals a

tender, palpable, right lower quadrant mass . There is no evidence of peritonitis or

systemic sepsis. Laboratory evaluation is remarkable for mild leukocytosis, and

abdominal ultrasound demonstrates an inflammatory reactions with suspicious

mass but no evidence of abscess. As the surgeon on call, your recommendation

would be.

A. Intravenous hydration, antibiotic prophylaxis, and urgent appendectomy

B. Intravenous hydration, antibiotics, bowel rest, and interval appendectomy

in 4 to 6 weeks.

C. Intravenous hydration, antibiotics, and appendectomy if no improvement in

12 to 24 hours

D. Intravenous hydration, antibiotics, and interval appendectomy when fever

has subsided, leukocyte count has returned to normal, and the patient is

pain free

E. Emergent obstetrical consultation for evaluation and treatment of possible

ectopic pregnancy.

Answer: A

The patient presented has a perforated appendix with a phlegmon, but no

abscess. One must routinely provide resuscitation and broad spectrum antiobiotic

coverage in this circumstance. As she is not systemically toxic, it would be

rational in a nonpregnant patient to treat this patient nonoperatively initially and

follow this with interval appendectomy.

However, in this circumstance, the risk of preterm labor associated with

anesthesia and pelvic inflammation increases with more advanced gestation, so

the best decision is to proceed with intravenous hydration, broad spectrum

antibiotic coverage and urgent appendectomy.

------------------------------------------------------------------------------------------------------------

93-Which of the following statements regarding laparoscopic and open

appendectomy is true:

Page 58: Q and a Exam for Seniors in general surgery

A. lap.app is associated with a shorter hospital stay by 36 hr.

B. lap.app is associated with decreased postoperative pain.

C. there is no difference in the return time to regular activity .

D. no difference in infection rates exist between the two techniques .

E. lap.app is the procedure of choice in all most all situations.

Answer B

With lap app.:

1. -less post op pain

2. -faster return to work

3. -but longer operation time.

4. -post op stayrarely reaches a difference of 24 hr.

5. -post op infection wound infection (less with lap.)

intraabdominal abscess (more with lap)

94-Which of the following statements about esophageal anatomy is correct?

A. The esophagus has a poor blood supply, which is segmental in distribution

and accounts for the high incidence of anastomotic leakage.

B. The esophageal serosa consists of a thin layer of fibroareolar tissue.

C. The esophagus has two distinct muscle layers, an outer, longitudinal one

and an inner, circular one, which are striated in the upper third and smooth

in the distal two thirds.

D. Injury to the recurrent laryngeal nerve results in vocal cord dysfunction but

does not affect swallowing.

E. The lymphatic drainage of the esophagus is relatively sparse, localized

primarily to adjacent paraesophageal lymph nodes.

Answer: C

Poor technique, not poor blood supply, explains most esophageal anastomotic

leaks. While the major blood supply of the esophagus is from four to six

segmental aortic esophageal arteries, there are extensive submucosal collaterals

from the inferior thyroid, intercostal, bronchial, inferior phrenic, and left gastric

arteries. The esophagus lacks serosa and instead is surrounded by mediastinal

connective tissue (adventitia). There are two muscle layers in the esophagus, an

outer longitudinal and an inner circular one. Both layers of the upper third of the

esophagus consist of striated muscle, while in the lower two thirds they are

(nonstriated) smooth muscle. The recurrent laryngeal branches of the vagus

nerves provide both parasympathetic innervation to the cervical esophagus and

innervation to the upper esophageal sphincter (UES). Injury to the recurrent

laryngeal nerve therefore results in improved UES function with secondary

aspiration on swallowing as well as vocal cord dysfunction and hoarseness. The

esophagus has extensive lymphatic drainage, with lymphatic capillaries coursing

longitudinally in the esophageal wall and communicating with paraesophageal,

paratracheal and subcarinal, other mediastinal, perigastric, and internal jugular

Page 59: Q and a Exam for Seniors in general surgery

lymph nodes. This accounts for the biologically aggressive nature of esophageal

carcinoma, which tends to metastasize early in its course.

95-Which of the following statements about gastric polyps is true?

A. Like their colonic counterparts, gastric epithelial polyps are common

tumors.

B. They are analogous to colorectal polyps in natural history.

C. Endoscopy can uniformly predict the histology of a polyp based on

location and appearance.

D. Gastric adenomatous polyps greater than 2 cm. in diameter should be

excised because of the risk of malignant transformation.

Answer: D

As early as 1895 Hauser reported an association between familial adenomatous

polyposis of the colon and multiple gastric polyps. This early association may

have given rise not only to the confusing nomenclature of gastric polyps but also

to the mistaken notion that they are analogous to colorectal polyps in microscopic

appearance and natural history. Unlike colonic polyps, gastric epithelial polyps

are very uncommon tumors (prevalence 0.4% to 0.8%).

Their histologic appearance cannot be predicted on the basis of location in the

stomach, although the endoscopic literature is beginning to define predictive

algorithms based on location and ultrasound. Multiple polyps are almost always

of a single histologic type. Gastric adenomatous polyps have long been

associated with adenocarcinoma. This association is directly related to the size of

the polyps. Up to 24% of polyps 2 cm. or greater in diameter are associated with

adenocarcinoma. In contrast, only 4% of polyps with a diameter less than 2 cm.

are associated with carcinoma. The risk, if any, of carcinoma in patients with

hyperplastic polyps appears to be associated with the atrophic gastritis that

frequently accompanies them rather than with the polyps themselves.

96-Metabolic complications of subtotal gastrectomy with Billroth I or Billroth II

reconstruction include all except:

A. Vit K and Vit B12 deficiency

B. Anemia.

C. Reactive hypoglycemia.

D. Dumping syndrome.

E. Metabolic bone disease.

Answer:A

Anemia develops in as many as 30% of patients within 15 years of surgery. The

cause is multifactorial and includes malabsorption of iron, folate, and vitamin B

12. A metabolic bone disease occurs in as many as 33% of patients, is similar to

osteomalacia, and is probably a result of malabsorption of calcium and vitamin D.

Reactive hypoglycemia occurs with rapid gastric emptying, resulting in increased

glucose absorption immediately after a meal. Initially there is hyperglycemia,

Page 60: Q and a Exam for Seniors in general surgery

leading to hyperinsulinemia and subsequent rapid glucose clearance and

symptomatic hypoglycemia. Dumping syndrome varies from very mild symptoms

to significantly disabling ones. The severe syndrome occurs in fewer than 5% of

patients. Small, frequent, dry meals of low osmolality reduces symptoms, and

somatostatin analog has been of some clinical use.

97-Which of the following patient scenarios would be best managed with anti-

reflux surgery?

A. A patient with heartburn but normal 24 hour pH monitoring and an intact

lower esophageal sphincter

B. A patient with primarily respiratory manifestations of gastroesophageal

reflux, without typical symptoms of GERD.

C. A patient with increased acid exposure and a mechanically defective

sphincter who responds well to medical therapy but requires continued

long-term medication for continued relief.

D. A patient with gastroesophageal reflux but excessive complaints of

epigastric pain, nausea, vomiting, and loss of appetite.

Answer: c

The first requirement for consideration of anti-reflux surgery is the objective

demonstration of the presence of GERD by 24-hour pH monitoring. Secondly, the

patient must have either symptoms or complications of the disease. Thirdly, the

disease should be caused by defect appropriate to surgical therapy, i.e., a

mechanically defective sphincter. Some patients with increased acid exposure

and a mechanically defective sphincter, and who have no complications of the

disease respond well to medical therapy, but they require long-term medication

for continued relief. These patients should be given the option of surgery as a

cost effective alternative.

Atypical symptoms of reflux such as respiratory manifestations often respond

well to anti-reflux surgery. When respiratory symptoms are combined with typical

symptoms such as heartburn and regurgitation, the results of anti-reflux surgery

are generally good.

Complaints of epigastric pain, nausea, vomiting, and loss of appetite may be due

to excessive duodenogastric reflux which occurs in about 11% of patients with

gastroesophageal reflux disease. This problem is usually, but not invariably,

confined to patients who have previous upper gastrointestinal surgery. The

coexistence of these gastric symptoms in a patient who also has typical

symptoms of GERD should prompt a thorough evaluation of the stomach using a

bile probe, 24 hour pH monitoring or radionucleotide scanning. In such patients,

the correction of only the incompetent cardia can result in a disgruntled individual

who continues to complain of nausea and epigastric pain on eating.

98-Barrett’s esophagus is a complication of gastroesophageal reflux disease.

Which of the following statements IS FALSE concerning this condition?

Page 61: Q and a Exam for Seniors in general surgery

A. The histologic hallmark is the presence of “specialized” columnar

epithelium regardless of how far it extends into the esophagus

B. Barrett’s epithelium will frequently regress with medical therapy or anti-

reflux surgery

C. High grade dysplasia will frequently be associated with foci of invasive

carcinoma

D. Patients with adenocarcinoma arising in Barrett’s esophagus have a high

incidence of p53 gene mutations.

Answer: B

Barrett’s esophagus is now recognized as a complication of advanced

gastroesophageal reflux disease. The histologic hallmark of Barrett’s esophagus

is the presence of “specialized” columnar epithelium, which shows features of

intestinal metaplasia, easily recognized by the presence of goblet cells. The

presence of specialized epithelium is now regarded as the pathonomonic feature

of Barrett’s esophagus regardless of how high it extends into the esophagus.

Barrett’s esophagus may exist on its own, or may be itself associated with

ulceration, stricture, and malignant change. Once Barrett’s epithelium is present,

medical therapy or anti-reflux surgery rarely causes it to regress. Unless it is

actually ablated, for example with laser therapy, it persists. The most significant

feature of Barrett’s esophagus is its malignant potential. The metaplastic

epithelium usually undergoes dysplastic change prior to becoming frankly

neoplastic. High grade dysplagia is synonymous with carcinoma in situ, and if the

esophagus is removed for such a condition, up to 50% will demonstrate foci of

invasive carcinoma.

In the past, the pathophysiology of Barrett’s esophagus was associated with

alkaline reflux on esophageal pH monitoring. However, more recently using a bile

sensor for monitoring bilirubin, this condition is frequently associated with

excessive bile in the esophagus. Repetitive injury from noxious gastric juice can

lead during the repair process to mutations in the p53 gene. Patients with

adenocarcinoma arising in Barrett’s esophagus have a high incidence of p53

mutations.

99-Which of the following statements is correct concerning the diagnostic studies

for esophageal carcinoma?

A. A chest and upper abdominal CT scan is useful for both staging and the

best test for predicting respectability.

B. A barium swallow is an unnecessary test in a patient with dysphagia

C. Bronchoscopy should be performed in all patients with carcinoma of the

upper but not lower two thirds of the esophagus

D. Bone and brain scans should be obtained routinely to rule out distant

metastasis

E. Endoscopic ultrasound is a potentially sensitive examination for the

staging of esophageal cancer.

Answer: E

Page 62: Q and a Exam for Seniors in general surgery

A barium swallow examination is the first study that should be obtained in a

patient who complains of dysphagia. Tumors of the cervical esophagus are

difficult to identify by barium swallow examination and carcinoma of the cardia

may be confused with achalasia or esophageal spasm. Nevertheless, the barium

swallow examination localizes obvious esophageal pathology in preparation for

subsequent esophagoscopy and allows the endoscopist to predict the level at

which the tumor is located and the area which requires the most careful

examination. The chest and upper abdominal CT scan is now the standard

radiographic technique for staging esophageal carcinoma. Esophageal wall

thickness, regional adenopathy or pulmonary, liver, adrenal or distant nodal

metastasis can be identified. Although CT is suggested to have a role in

evaluating resectability of esophageal carcinoma, it is particularly limited in its

ability to detect invasion of the gastric cardia or aortic invasion. Bone scan is not

warranted unless the patient has specific complaints suggesting that bone

metastases exists. Similarly, routine brain scans are not indicated as brain

metastases from carcinoma of the esophagus are uncommon. Bronchoscopy

should be performed in patients with carcinoma of the upper and middle thirds of

the esophagus to exclude invasion of the posterior membranous trachea or

mainstem bronchi, which precludes a safe esophagectomy. Endoscopic

ultrasound is being used with increasing frequency as an adjunct to the standard

radiologic and endoscopic assessment of esophageal disease. It offers the

potential for more sensitive staging of esophageal carcinoma by detecting the

depth of invasion and the presence of abnormal mediastinal adenopathy.

100-Which of the following statements regarding human gastric acid secretion is

FALSE?

A. Fasting acid secretion, normally 2 to 5 mEq/h, is due to ambient vagal tone

and histamine secretion

B. Truncal vagotomy decreases basal secretion by 80%

C. Histamine2 receptor antagonist administration can decrease basal acid

secretion by 80%

D. Fasting acid secretion, normally 5 to 10 mEq/h, is due to circulating levels

of gastrin.

Answer: D

Both vagal tone and locally secreted histamine are presumed to be the

determinants of basal acid secretion in humans. Gastrin does not have a role in

basal acid secretion in normal individuals.

Parietal cell activation and the resultant acid secretory response is greater to a

combination of agonists than the sum of the responses to the agents used singly.

This increase in responsiveness is termed potentiation. Potentiating interactions

are most apparent when the stimulants use different second messenger systems,

for example, acetylcholine and histamine. Conversely, blockade of receptors to

one stimulant also blocks responsiveness to the other agonist. Because of this

interaction, blockade of histamine receptors by agents like cimetidine decreases

Page 63: Q and a Exam for Seniors in general surgery

responsiveness to acetylcholine. Blockage of acetylcholine release by vagotomy

decreases responsiveness to histamine secreted by gastric mast cells.

Both vagotomy and histamine2 receptor antagonists decrease basal acid

secretion by approximately 80%.

With my best wishes

DR.JABER ALFAIFI

[email protected]